THMMY.gr

Μαθήματα Βασικού Κύκλου => Συστήματα Αυτομάτου Ελέγχου II - Παλιά Θέματα => Topic started by: adianohtos on June 27, 2015, 16:03:31 pm



Title: [ΣΑΕ ΙΙ] Θέματα 2015
Post by: adianohtos on June 27, 2015, 16:03:31 pm
Καποιο απο τα θεματα 1,4 και 5 φεβρουαριου 2015 κανεις;


Title: Re: [ΣΑΕ ΙΙ] Θέματα 2015
Post by: Captain Mixos on June 27, 2015, 19:29:11 pm
Καποιο απο τα θεματα 1,4 και 5 φεβρουαριου 2015 κανεις;

+1


Title: Re: [ΣΑΕ ΙΙ] Θέματα 2015
Post by: ripperj on June 29, 2015, 14:06:28 pm
Φεβρουαριος 2015 θεμα 1, παιρνουμε οτι ειναι block διαγωνιος και ο κάθε υποπίνακας Αi=[σi ωi ; -ωi σi] και οι ιδιοτιμές ειναι λi=σi +/- jλi, βγαινουν 3 σi, -1 -2 και 0, άρα έχουμε (απλή) ευστάθεια.

Αν ήταν ολα αρνητικά, θα ηταν ασυμπτωτικά ευσταθές (μαλλον)


Title: Re: [ΣΑΕ ΙΙ] Θέματα 2015
Post by: ripperj on June 29, 2015, 14:31:20 pm
μια διευκρινιση για το θεμα 4, φεβρουαριος 2015

αδιασταυρωτες πληροφοριες θελουν τα χ1(0)=χ01 και χ2(0)=χ02 ειναι το σημειο ισορροπίας, άρα πρεπει να κανουμε αλλαγη μεταβλητης για να βρουμε τους οριακους κυκλους στο ερωτημα α.

στο β, οταν λεει για φραγμενες τροχιες, μαλλον θελει να αποδειξουμε οτι το συστημα ειναι ελέγξιμο, άρα απο οποιοδηποτε σημειο μπορουμε να φτασουμε στο Σ.Ι. σε πεπερασμενο χρόνο, άρα μέσω φραγμένης τροχιας. οποτε αν το δεχτουμε αυτο, η τροχια x1(t;x01) ειναι απο ενα τυχαιο t μέχρι το x01, το οποιο θα ειναι το Σ.Ι., που μαλλον επαληθευει την παραπανω αδιευκρινιστη πληροφορια για τα x01 x02..

any thoughts?

---
βεβαια για να δειχτει η ελεγξιμοτητα, πρεπει να γραμικοποιηθει το συστημα, οποτε γινεται της παλαβης :P


Title: Re: [ΣΑΕ ΙΙ] Θέματα 2015
Post by: forisonex on June 29, 2015, 14:47:39 pm
Υπάρχουν εκφωνήσεις των θεμάτων Προόδου 2015?


Title: Re: [ΣΑΕ ΙΙ] Θέματα 2015
Post by: Σαλτιμπάγκος on June 29, 2015, 23:24:48 pm
μια διευκρινιση για το θεμα 4, φεβρουαριος 2015

αδιασταυρωτες πληροφοριες θελουν τα χ1(0)=χ01 και χ2(0)=χ02 ειναι το σημειο ισορροπίας, άρα πρεπει να κανουμε αλλαγη μεταβλητης για να βρουμε τους οριακους κυκλους στο ερωτημα α.

χρειαζεται να κανουμε την αλλαγη μεταβλητης; αν χρησιμοποιησεις απλα τα χ1 κ χ2 μια χαρα δεν βγαινει οπως σελ 21 του pdf?


Title: Re: [ΣΑΕ ΙΙ] Θέματα 2015
Post by: vlad on June 30, 2015, 00:57:25 am
ΛΕΠΟΝ.Θεμα 5 Φλεβαρης 15. Λεει το αλανι οτι M(q).... γνωστα. Δηλαδη σταθερες;Ή γνωστες οι συναρτησεις;


Title: Re: [ΣΑΕ ΙΙ] Θέματα 2015
Post by: svart on June 30, 2015, 01:02:36 am
γνωστά πάει να πει οτι μπορείς να τα χρησιμοποιήσεις στον ελεγκτή σου. αν δεν σου λεει ότι ενα μέγεθος είναι γνωστό ή μετρήσιμο δεν μπορείς να το βάλεις με σιγουριά


Title: Re: [ΣΑΕ ΙΙ] Θέματα 2015
Post by: vlad on June 30, 2015, 01:04:51 am
γνωστά πάει να πει οτι μπορείς να τα χρησιμοποιήσεις στον ελεγκτή σου. αν δεν σου λεει ότι ενα μέγεθος είναι γνωστό ή μετρήσιμο δεν μπορείς να το βάλεις με σιγουριά
Δηλαδη μπορω να τα παρω σαν σταθερες ετσι;Τα Μ,f,Κ. (btw το εχεις λυσει το θεμα; )


Title: Re: [ΣΑΕ ΙΙ] Θέματα 2015
Post by: svart on June 30, 2015, 01:13:36 am
δεν είναι σταθερές, αλλά συναρτήσεις ή εστω μεταβλητά μεγέθη. έχουμε και λέμε

το σύστημα του σχήματος είναι της μορφής

Tex code

θεωρώ

Tex code => Tex code
M > 0 άρα αντιστρέψιμος =>  Tex code

Τέλος θεωρώ
Tex code


Title: Re: [ΣΑΕ ΙΙ] Θέματα 2015
Post by: vlad on June 30, 2015, 01:23:35 am
Tex code
Μπορεις να εξηγησεις πως προκυπτει αυτο αν δεν σου ειναι δυσκολο;


Title: Re: [ΣΑΕ ΙΙ] Θέματα 2015
Post by: svart on June 30, 2015, 01:27:29 am
το υ είναι προσωρινό για διευκόλυνση των πράξεων. θεωρώ το συγκεκριμένο u, για να φύγουν οι συγκεκριμένοι όροι από τη δοσμένη εξίσωση και να τους αντικαταστήσω με αυτούς που θέλω εγώ μέσω του υ


Title: Re: [ΣΑΕ ΙΙ] Θέματα 2015
Post by: Give_Life_Back on August 07, 2015, 15:55:36 pm
Έχει κανείς  ενδεικτικές λύσεις για τα θέματα Ιουνίου 2015?


Title: Re: [ΣΑΕ ΙΙ] Θέματα 2015
Post by: c0ndemn3d on August 07, 2015, 17:41:04 pm
A) Λάθος
Β) Λάθος
Γ) Λάθος
Δ) Σωστό
Ε) Λάθος

Θέμα 2
---------

α) Θέσε τις παραγώγους ίσες με 0 και βρες το σημείο ισορροπίας

β) Εφαρμόζεις τους τύπους για γραμμικοποίηση και δείχνεις ότι οι ιδιοτιμές του πίνακα έχουν αρνητικό πραγματικό μέρος. Δικαιολογείς ότι η ευστάθεια είναι τοπική καθώς το γραμμικοποιημένο σύστημα είναι μια προσέγγιση του κανονικού.

Θέμα 3
---------

Θα πάρεις την κλασική υποψήφια Lyapunov. Θα την βγάλεις μικρότερη ίση του 0 με πρόσημο ανεξάρτητο του t λόγω της ανισότητας που σου δίνει. Τότε θα πας σε Lasalle και θα το κάνεις όπως στα παραδείγματα του Ροβιθάκη.

Θέμας 4
----------
α) Εφαρμόζεις τύπους
β) Παίρνεις τον πίνακα ελεγξιμότης και πρέπει να βγάλεις ορίζουσα διάφορη του 0
γ) Θα πρέπει να το φέρεις ώστε το (0,0) να είναι το ΣΙ. Μετά, επειδή δεν ξέρεις τις παραμέτρους του συστήματος, αναγκαστικά θα πας με δυναμική ανάδραση και θα πρέπει να χρησιμοποιήσεις τις ανισότητες που σου δίνει ώστε 1) το σύστημα να είναι ευσταθές 2) να μην χρησιμοποιείς άγνωστα μεγέθη στον ελεγκτή σου.


Title: Re: [ΣΑΕ ΙΙ] Θέματα 2015
Post by: quino on August 11, 2015, 20:22:48 pm
Για το ερώτημα α του θέματος 4 του Ιουνίου του '15, δεδομένου ότι μας δίνει είσοδο  θα πρέπει να επιλέξουμε και για αυτήν μία τιμή για να έχουμε συγκεκριμένο σημείο ισορροπίας ? Σε ένα  παράδειγμα στις σημειώσεις  ζητάει να βρεθεί η τιμή της εισόδου για να έχουμε  Σ.Ι το (0,0).  :???:


Title: Re: [ΣΑΕ ΙΙ] Θέματα 2015
Post by: svart on August 11, 2015, 20:38:33 pm
Για το ερώτημα α του θέματος 4 του Ιουνίου του '15, δεδομένου ότι μας δίνει είσοδο  θα πρέπει να επιλέξουμε και για αυτήν μία τιμή για να έχουμε συγκεκριμένο σημείο ισορροπίας ? Σε ένα  παράδειγμα στις σημειώσεις  ζητάει να βρεθεί η τιμή της εισόδου για να έχουμε  Σ.Ι το (0,0).  :???:

Θεωρείς το ΣΙ το (x*, u*) και εφράζεις το u* συναρτήσει του x*, οπότε το ΣΙ σου θα είναι (x*, u*(x*))


Title: Re: [ΣΑΕ ΙΙ] Θέματα 2015
Post by: vasilis94 on August 11, 2015, 20:44:33 pm
Για το ερώτημα α του θέματος 4 του Ιουνίου του '15, δεδομένου ότι μας δίνει είσοδο  θα πρέπει να επιλέξουμε και για αυτήν μία τιμή για να έχουμε συγκεκριμένο σημείο ισορροπίας ? Σε ένα  παράδειγμα στις σημειώσεις  ζητάει να βρεθεί η τιμή της εισόδου για να έχουμε  Σ.Ι το (0,0).  :???:

Θεωρείς το ΣΙ το (x*, u*) και εφράζεις το u* συναρτήσει του x*, οπότε το ΣΙ σου θα είναι (x*, u*(x*))

+1, δες το παράδειγμα σελίδες 31-32 exomag

Αν βάλεις συγκεκριμένη είσοδο, έφυγε τζάμπα μηδενισμός με τη μία...


Title: Re: [ΣΑΕ ΙΙ] Θέματα 2015
Post by: svart on August 11, 2015, 20:46:28 pm
Για το ερώτημα α του θέματος 4 του Ιουνίου του '15, δεδομένου ότι μας δίνει είσοδο  θα πρέπει να επιλέξουμε και για αυτήν μία τιμή για να έχουμε συγκεκριμένο σημείο ισορροπίας ? Σε ένα  παράδειγμα στις σημειώσεις  ζητάει να βρεθεί η τιμή της εισόδου για να έχουμε  Σ.Ι το (0,0).  :???:

Θεωρείς το ΣΙ το (x*, u*) και εφράζεις το u* συναρτήσει του x*, οπότε το ΣΙ σου θα είναι (x*, u*(x*))

+1, δες το παράδειγμα σελίδες 31-32 exomag

Αν βάλεις συγκεκριμένη είσοδο, έφυγε τζάμπα μηδενισμός με τη μία...

Επίσης νομίζω θεωρεί λάθος το να εκφράσεις το x* συναρτήσει του u*


Title: Re: [ΣΑΕ ΙΙ] Θέματα 2015
Post by: quino on August 11, 2015, 20:50:01 pm
Ευχαριστώ πολύ !!   :)


Title: Re: [ΣΑΕ ΙΙ] Θέματα 2015
Post by: nikos1 on August 20, 2015, 10:51:15 am
Αν μπορει καποιος ας ποσταρει τις λυσεις των θεματων 4,5 Φεβρουαριου 2015


Title: Re: [ΣΑΕ ΙΙ] Θέματα 2015
Post by: giwrgosbg on August 23, 2015, 15:09:19 pm
Αν μπορει καποιος ας ποσταρει τις λυσεις των θεματων 4,5 Φεβρουαριου 2015

+1


Title: Re: [ΣΑΕ ΙΙ] Θέματα 2015
Post by: giwrgosbg on August 26, 2015, 14:38:19 pm
A) Λάθος
Β) Λάθος
Γ) Λάθος
Δ) Σωστό
Ε) Λάθος

Θέμα 2
---------

α) Θέσε τις παραγώγους ίσες με 0 και βρες το σημείο ισορροπίας

β) Εφαρμόζεις τους τύπους για γραμμικοποίηση και δείχνεις ότι οι ιδιοτιμές του πίνακα έχουν αρνητικό πραγματικό μέρος. Δικαιολογείς ότι η ευστάθεια είναι τοπική καθώς το γραμμικοποιημένο σύστημα είναι μια προσέγγιση του κανονικού.

Θέμα 3
---------

Θα πάρεις την κλασική υποψήφια Lyapunov. Θα την βγάλεις μικρότερη ίση του 0 με πρόσημο ανεξάρτητο του t λόγω της ανισότητας που σου δίνει. Τότε θα πας σε Lasalle και θα το κάνεις όπως στα παραδείγματα του Ροβιθάκη.

Θέμας 4
----------
α) Εφαρμόζεις τύπους
β) Παίρνεις τον πίνακα ελεγξιμότης και πρέπει να βγάλεις ορίζουσα διάφορη του 0
γ) Θα πρέπει να το φέρεις ώστε το (0,0) να είναι το ΣΙ. Μετά, επειδή δεν ξέρεις τις παραμέτρους του συστήματος, αναγκαστικά θα πας με δυναμική ανάδραση και θα πρέπει να χρησιμοποιήσεις τις ανισότητες που σου δίνει ώστε 1) το σύστημα να είναι ευσταθές 2) να μην χρησιμοποιείς άγνωστα μεγέθη στον ελεγκτή σου.

.


θεμα 4 το γ μπορει κάποιος να ανεβάσει λύση γιατί με β'αση  το πχ που μας είχε κανει ο Robi νομίζω κατι δεν εφαρμόζω σωστα  :-\


Title: Re: [ΣΑΕ ΙΙ] Θέματα 2015
Post by: lady_of_winter on September 07, 2015, 15:13:07 pm
το πρώτο σ-λ του ιούνη 2015 γιατί είναι λάθος?
γενικά είχε αναφερθεί σε πόλους συνάρτησης μεταφοράς? ::)


Title: Re: [ΣΑΕ ΙΙ] Θέματα 2015
Post by: c0ndemn3d on September 07, 2015, 15:18:22 pm
το πρώτο σ-λ του ιούνη 2015 γιατί είναι λάθος?
γενικά είχε αναφερθεί σε πόλους συνάρτησης μεταφοράς? ::)

Κοίτα στην αποδόμηση Kalman. Ένα σύστημα αναλύεται σε 4 υποσυστήματα

1) Ένα ελέγξιμο - παρατηρήσιμο.
2) Ένα μη-ελέγξιμο - παρατηρήσιμο.
3) Ένα ελέγξιμο - μη-παρατηρήσιμο.
4) Ένα μη-ελέγξιμο - μη-παρατηρήσιμο.


Αποδεικνύεται ότι η συνάρτηση μεταφοράς ενός τέτοιου συστήματος είναι ταυτόσημη με τη συνάρτηση μεταφοράς του ελέγξιμου και παρατηρήσιμου υποσυστήματος, άρα οι ιδιοτιμές των άλλων 3 υποσυστημάτων ΔΕΝ είναι πόλοι του συστήματος.


Title: Re: [ΣΑΕ ΙΙ] Θέματα 2015
Post by: lady_of_winter on September 07, 2015, 15:21:58 pm
να σαι καλα :)
δεν παιζει να το σκεφτομουν αυτο


Title: Re: [ΣΑΕ ΙΙ] Θέματα 2015
Post by: c0ndemn3d on September 07, 2015, 15:26:44 pm
να σαι καλα :)
δεν παιζει να το σκεφτομουν αυτο

Ίσως ούτε εμείς, αλλά ευτυχώς το είχε πει ο Ρόβι 2-3 φορές το συγκεκριμένο Σ-Λ μέσα στην τάξη και το θυμόμουν


Title: Re: [ΣΑΕ ΙΙ] Θέματα 2015
Post by: Μεταλλαγμένη Πάπια on September 09, 2015, 18:58:47 pm
Φεβρουαριος 2015, θεμα 4 β κανεις; Κατι εχω σκεφτει αλλα δεν ειμαι ιδιαιτερα σιγουρος...


Title: Re: [ΣΑΕ ΙΙ] Θέματα 2015
Post by: iliachry on September 10, 2015, 16:15:48 pm
Φεβρουαριος 2015, θεμα 4 β κανεις; Κατι εχω σκεφτει αλλα δεν ειμαι ιδιαιτερα σιγουρος...

η λύση υπάρχει στο 5ο κεφάλαιο από τις σημειώσεις του Ροβιθάκη(κάτω από το θεώρημα 5.1 στην παρατήρηση).



Title: Re: [ΣΑΕ ΙΙ] Θέματα 2015
Post by: Μεταλλαγμένη Πάπια on September 10, 2015, 18:40:12 pm
Εεεε... σίγουρα; Εγώ το μόνο που βλέπω στην παρατήρηση εκείνη είναι ενα σχόλιο για την ελεγξιμότητα... Γενικά αν υπήρχε λυμένη αυτή η άσκηση απο ροβιθάκη, μάλλον θα ήταν σε άλλο κεφάλαιο, αυτό το θέμα δεν έχει σχέση με ελεγξιμότητα ( νομίζω δηλαδή  :P )

Edit: Τέλως πάντων, εγω θα ποσταρω αυτο που σκεφτηκα. Εμενα σωστο μου φαινεται, αλλα εγω ειμαι χαζος γενικα, οποτε πειτε και σεις...

Δινεται α>0 και β<0. Επισης ξερουμε οτι η συναρτηση lyapunov μπορει να ειναι οποιαδηποτε αρκει να ειναι θετικα ημιορισμενη σωστα; Επιλεγω την εξης:

V = 1/2α * χ12 - 1/2β * χ22 , η οποια ειναι θετικα ημιορισμενη διοτι β<0 και μαζι με το - που βαλαμε ειναι οκ...

Υπολογιζουμε την παραγωγο και μετα απο πραξεις εγω καταληγω σε αυτο:

 .
V = 1/αβ * ( βχ12 - αχ22 ) * ( 4 - χ12 - χ22 )

  • Το 1/αβ ειναι <0 παντα και παντου
  • Το βχ12 - αχ22 ειναι συνολικα <=0 ( =0 στο (0,0) ) γιατι αυτο ισχυει για καθε ορο, ειναι σαν αθροισμα τετραγωνων με ενα - απεξω
  • Ο τελευταιος ορος ειναι που αλλαζει προσημο αναλογα με το αν βρισκεται μεσα η εξω απο τον κυκλο

Οποτε ουσιαστικα εχουμε τις εξης περιπτωσεις

  • χ12 + χ22 < 4 . Τοτε η παραγωγος ειναι (-) * (-) * (+) = (+) οποτε οι καταστασεις θα κινουνται προς τον κυκλο μεχρι να τον φτασουν. Οταν φτασουν θα κινουνται πανω του.
  • χ12 + χ22 > 4 . Τοτε η παραγωγος ειναι (-) * (-) * (-) = (-) οποτε οι καταστασεις ξανα θα κινουνται προς τον κυκλο και οταν φτασουν θα κινουνται πανω του.

Με βαση αυτα καταληγω οτι ανεξαρτητα απο σημειο εκκινησης οι καταστασεις τελικα θα κινουνται πανω στον κυκλο με κεντρο (0,0) και ακτινα 2. Τι λετε;


Title: Re: [ΣΑΕ ΙΙ] Θέματα 2015
Post by: iliachry on September 10, 2015, 20:30:25 pm
Εεεε... σίγουρα; Εγώ το μόνο που βλέπω στην παρατήρηση εκείνη είναι ενα σχόλιο για την ελεγξιμότητα... Γενικά αν υπήρχε λυμένη αυτή η άσκηση απο ροβιθάκη, μάλλον θα ήταν σε άλλο κεφάλαιο, αυτό το θέμα δεν έχει σχέση με ελεγξιμότητα ( νομίζω δηλαδή  :P )

Edit: Τέλως πάντων, εγω θα ποσταρω αυτο που σκεφτηκα. Εμενα σωστο μου φαινεται, αλλα εγω ειμαι χαζος γενικα, οποτε πειτε και σεις...

Δινεται α>0 και β<0. Επισης ξερουμε οτι η συναρτηση lyapunov μπορει να ειναι οποιαδηποτε αρκει να ειναι θετικα ημιορισμενη σωστα; Επιλεγω την εξης:

V = 1/2α * χ12 - 1/2β * χ22 , η οποια ειναι θετικα ημιορισμενη διοτι β<0 και μαζι με το - που βαλαμε ειναι οκ...

Υπολογιζουμε την παραγωγο και μετα απο πραξεις εγω καταληγω σε αυτο:

 .
V = 1/αβ * ( βχ12 - αχ22 ) * ( 4 - χ12 - χ22 )

  • Το 1/αβ ειναι <0 παντα και παντου
  • Το βχ12 - αχ22 ειναι συνολικα <=0 ( =0 στο (0,0) ) γιατι αυτο ισχυει για καθε ορο, ειναι σαν αθροισμα τετραγωνων με ενα - απεξω
  • Ο τελευταιος ορος ειναι που αλλαζει προσημο αναλογα με το αν βρισκεται μεσα η εξω απο τον κυκλο

Οποτε ουσιαστικα εχουμε τις εξης περιπτωσεις

  • χ12 + χ22 < 4 . Τοτε η παραγωγος ειναι (-) * (-) * (+) = (+) οποτε οι καταστασεις θα κινουνται προς τον κυκλο μεχρι να τον φτασουν. Οταν φτασουν θα κινουνται πανω του.
  • χ12 + χ22 > 4 . Τοτε η παραγωγος ειναι (-) * (-) * (-) = (-) οποτε οι καταστασεις ξανα θα κινουνται προς τον κυκλο και οταν φτασουν θα κινουνται πανω του.

Με βαση αυτα καταληγω οτι ανεξαρτητα απο σημειο εκκινησης οι καταστασεις τελικα θα κινουνται πανω στον κυκλο με κεντρο (0,0) και ακτινα 2. Τι λετε;

ναι μπερδεύτηκα. Άλλο θέμα νόμιζα ρωτούσες!
όσον αφορά την λύση σου στο 4β νομίζω είναι σωστή.


Title: Re: [ΣΑΕ ΙΙ] Θέματα 2015
Post by: SportBillyPap on September 11, 2015, 12:53:33 pm
Εεεε... σίγουρα; Εγώ το μόνο που βλέπω στην παρατήρηση εκείνη είναι ενα σχόλιο για την ελεγξιμότητα... Γενικά αν υπήρχε λυμένη αυτή η άσκηση απο ροβιθάκη, μάλλον θα ήταν σε άλλο κεφάλαιο, αυτό το θέμα δεν έχει σχέση με ελεγξιμότητα ( νομίζω δηλαδή  :P )

Edit: Τέλως πάντων, εγω θα ποσταρω αυτο που σκεφτηκα. Εμενα σωστο μου φαινεται, αλλα εγω ειμαι χαζος γενικα, οποτε πειτε και σεις...

Δινεται α>0 και β<0. Επισης ξερουμε οτι η συναρτηση lyapunov μπορει να ειναι οποιαδηποτε αρκει να ειναι θετικα ημιορισμενη σωστα; Επιλεγω την εξης:

V = 1/2α * χ12 - 1/2β * χ22 , η οποια ειναι θετικα ημιορισμενη διοτι β<0 και μαζι με το - που βαλαμε ειναι οκ...

Υπολογιζουμε την παραγωγο και μετα απο πραξεις εγω καταληγω σε αυτο:

 .
V = 1/αβ * ( βχ12 - αχ22 ) * ( 4 - χ12 - χ22 )

  • Το 1/αβ ειναι <0 παντα και παντου
  • Το βχ12 - αχ22 ειναι συνολικα <=0 ( =0 στο (0,0) ) γιατι αυτο ισχυει για καθε ορο, ειναι σαν αθροισμα τετραγωνων με ενα - απεξω
  • Ο τελευταιος ορος ειναι που αλλαζει προσημο αναλογα με το αν βρισκεται μεσα η εξω απο τον κυκλο

Οποτε ουσιαστικα εχουμε τις εξης περιπτωσεις

  • χ12 + χ22 < 4 . Τοτε η παραγωγος ειναι (-) * (-) * (+) = (+) οποτε οι καταστασεις θα κινουνται προς τον κυκλο μεχρι να τον φτασουν. Οταν φτασουν θα κινουνται πανω του.
  • χ12 + χ22 > 4 . Τοτε η παραγωγος ειναι (-) * (-) * (-) = (-) οποτε οι καταστασεις ξανα θα κινουνται προς τον κυκλο και οταν φτασουν θα κινουνται πανω του.

Με βαση αυτα καταληγω οτι ανεξαρτητα απο σημειο εκκινησης οι καταστασεις τελικα θα κινουνται πανω στον κυκλο με κεντρο (0,0) και ακτινα 2. Τι λετε;
Η σκεψη σου ειναι σιγουρα σωστη.Απλα νομιζω οτι ο αγαπητος ροβι δεν ειχε αυτο στο μυαλο του αλλα μαλλον κατι με ολοκληρωματα και ορια..Κανεις δεν ξερει :P


Title: Re: [ΣΑΕ ΙΙ] Θέματα 2015
Post by: iliachry on September 11, 2015, 19:28:46 pm
Εεεε... σίγουρα; Εγώ το μόνο που βλέπω στην παρατήρηση εκείνη είναι ενα σχόλιο για την ελεγξιμότητα... Γενικά αν υπήρχε λυμένη αυτή η άσκηση απο ροβιθάκη, μάλλον θα ήταν σε άλλο κεφάλαιο, αυτό το θέμα δεν έχει σχέση με ελεγξιμότητα ( νομίζω δηλαδή  :P )

Edit: Τέλως πάντων, εγω θα ποσταρω αυτο που σκεφτηκα. Εμενα σωστο μου φαινεται, αλλα εγω ειμαι χαζος γενικα, οποτε πειτε και σεις...

Δινεται α>0 και β<0. Επισης ξερουμε οτι η συναρτηση lyapunov μπορει να ειναι οποιαδηποτε αρκει να ειναι θετικα ημιορισμενη σωστα; Επιλεγω την εξης:

V = 1/2α * χ12 - 1/2β * χ22 , η οποια ειναι θετικα ημιορισμενη διοτι β<0 και μαζι με το - που βαλαμε ειναι οκ...

Υπολογιζουμε την παραγωγο και μετα απο πραξεις εγω καταληγω σε αυτο:

 .
V = 1/αβ * ( βχ12 - αχ22 ) * ( 4 - χ12 - χ22 )

  • Το 1/αβ ειναι <0 παντα και παντου
  • Το βχ12 - αχ22 ειναι συνολικα <=0 ( =0 στο (0,0) ) γιατι αυτο ισχυει για καθε ορο, ειναι σαν αθροισμα τετραγωνων με ενα - απεξω
  • Ο τελευταιος ορος ειναι που αλλαζει προσημο αναλογα με το αν βρισκεται μεσα η εξω απο τον κυκλο

Οποτε ουσιαστικα εχουμε τις εξης περιπτωσεις

  • χ12 + χ22 < 4 . Τοτε η παραγωγος ειναι (-) * (-) * (+) = (+) οποτε οι καταστασεις θα κινουνται προς τον κυκλο μεχρι να τον φτασουν. Οταν φτασουν θα κινουνται πανω του.
  • χ12 + χ22 > 4 . Τοτε η παραγωγος ειναι (-) * (-) * (-) = (-) οποτε οι καταστασεις ξανα θα κινουνται προς τον κυκλο και οταν φτασουν θα κινουνται πανω του.

Με βαση αυτα καταληγω οτι ανεξαρτητα απο σημειο εκκινησης οι καταστασεις τελικα θα κινουνται πανω στον κυκλο με κεντρο (0,0) και ακτινα 2. Τι λετε;
Η σκεψη σου ειναι σιγουρα σωστη.Απλα νομιζω οτι ο αγαπητος ροβι δεν ειχε αυτο στο μυαλο του αλλα μαλλον κατι με ολοκληρωματα και ορια..Κανεις δεν ξερει :P


πως μπορεί να λυθεί με ολοκληρώματα και όρια;


Title: Re: [ΣΑΕ ΙΙ] Θέματα 2015
Post by: SportBillyPap on September 12, 2015, 11:14:06 am
Δε ξέρω και εγώ την παραπάνω λύση θα ακολουθουσα απλά νομίζω ότι ο ίδιος θα το έπαιρνε λαθος


Title: Re: [ΣΑΕ ΙΙ] Θέματα 2015
Post by: SportBillyPap on September 12, 2015, 18:08:00 pm
αμα καποιος εχει καταφερει να λυσει το θεμα 4γ) του ιουνη του 2015 αν μπορει να το ανεβασει  :'(  ;D


Title: Re: [ΣΑΕ ΙΙ] Θέματα 2015
Post by: Perasmus on September 13, 2015, 01:45:29 am
αμα καποιος εχει καταφερει να λυσει το θεμα 4γ) του ιουνη του 2015 αν μπορει να το ανεβασει  :'(  ;D


++1  ;D


Title: Re: [ΣΑΕ ΙΙ] Θέματα 2015
Post by: quino on September 13, 2015, 16:43:37 pm
Στο θέμα 5 του Φεβρουαρίου η εξωτερική δυναμη στο επιθυμητο συστημα F ειναι αγνωστο μεγεθος μιας και δε μας λεει κατι στην εκφωνηση ?  Φανταζομαι οτι ειναι κατι που αλλαζει
συνεπως δεν μπορεί να χρησιμοποιηθεί στην έκφραση για το u ?


Title: Re: [ΣΑΕ ΙΙ] Θέματα 2015
Post by: Conflict on September 13, 2015, 21:10:37 pm
Στο θέμα 5 του Φεβρουαρίου η εξωτερική δυναμη στο επιθυμητο συστημα F ειναι αγνωστο μεγεθος μιας και δε μας λεει κατι στην εκφωνηση ?  Φανταζομαι οτι ειναι κατι που αλλαζει
συνεπως δεν μπορεί να χρησιμοποιηθεί στην έκφραση για το u ?

Εγω νομιζω πως το F ταυτιζεται με το u μιας και τα δυο αναφερονται στην "εξωτερικη δυναμη διεγερσης" συμφωνα με την εκφωνηση.


Title: Re: [ΣΑΕ ΙΙ] Θέματα 2015
Post by: c0ndemn3d on September 13, 2015, 21:15:51 pm
Στο θέμα 5 του Φεβρουαρίου η εξωτερική δυναμη στο επιθυμητο συστημα F ειναι αγνωστο μεγεθος μιας και δε μας λεει κατι στην εκφωνηση ?  Φανταζομαι οτι ειναι κατι που αλλαζει
συνεπως δεν μπορεί να χρησιμοποιηθεί στην έκφραση για το u ?

Εγω νομιζω πως το F ταυτιζεται με το u μιας και τα δυο αναφερονται στην "εξωτερικη δυναμη διεγερσης" συμφωνα με την εκφωνηση.


Μην κολλάτε, είσοδος του συστήματος είναι η δύναμη F, απλά γενικά η είσοδος συμβολίζεται με u.


Title: Re: [ΣΑΕ ΙΙ] Θέματα 2015
Post by: quino on September 13, 2015, 21:46:26 pm
Στο θέμα 5 του Φεβρουαρίου η εξωτερική δυναμη στο επιθυμητο συστημα F ειναι αγνωστο μεγεθος μιας και δε μας λεει κατι στην εκφωνηση ?  Φανταζομαι οτι ειναι κατι που αλλαζει
συνεπως δεν μπορεί να χρησιμοποιηθεί στην έκφραση για το u ?

Εγω νομιζω πως το F ταυτιζεται με το u μιας και τα δυο αναφερονται στην "εξωτερικη δυναμη διεγερσης" συμφωνα με την εκφωνηση.

Ουσιαστικά η απορία μου είναι αν στην έκφραση για το u μπορεί να χρησιμοποιηθεί το F .. Θα μπορούσε το u να ειναι κατι της μορφής :
(http://)
Μην κολλάτε, είσοδος του συστήματος είναι η δύναμη F, απλά γενικά η είσοδος συμβολίζεται με u.
Οκ...Βασικά αυτό που ρωτάω είναι αν η είσοδος ελέγχου θα μπορούσε να είναι κάτι της μορφής :
(https://scontent-mxp1-1.xx.fbcdn.net/hphotos-xpt1/v/t34.0-12/12025300_1060118114021625_2093928719_n.jpg?oh=16529ff717b2cc3fd8e5cf59e381fcf2&oe=55F794EA)
και αν θα πρέπει να αντικαταστήσουμε με μια γνωστή έκφραση και το q'' ... Προσωπικά μου φαίνεται πολύ περίπλοκο, αν την έχει λύσει κάποιος ας πει τη λύση  :D


Title: Re: [ΣΑΕ ΙΙ] Θέματα 2015
Post by: tsakos on September 14, 2015, 13:44:35 pm
Με μεγάλη επιφύλαξη επισυναπτω τη λύση του 4ου θέματος Ιούνιος 15. Αν δεν είναι σωστό πείτε προς αποφυγήν αυτού..


Title: Re: [ΣΑΕ ΙΙ] Θέματα 2015
Post by: ALEO on September 14, 2015, 13:48:49 pm
Στο Θέμα 3 του ιουνίου 2015, πως αποδεικνύουμε την ολική ασυμπτωτική ευστάθεια?? Σε προηγούμενο post o c0ndemn3d λέει οτι αφού βρούμε οτι η παράγωγος της Lyapunov είναι αρνητικά ημιορισμένη μετά κάνουμε Lasalle. Με το Lasalle ομως βρίσκουμε τοπική ασυμπτωτική ευστάθεια ενώ εμείς θέλουμε ολική.. Ξέρει κανείς??


Title: Re: [ΣΑΕ ΙΙ] Θέματα 2015
Post by: SportBillyPap on September 14, 2015, 13:55:27 pm
Με μεγάλη επιφύλαξη επισυναπτω τη λύση του 4ου θέματος Ιούνιος 15. Αν δεν είναι σωστό πείτε προς αποφυγήν αυτού..
πως δειχνεις οτι p1<a/c p2<1/c


Title: Re: [ΣΑΕ ΙΙ] Θέματα 2015
Post by: tsakos on September 14, 2015, 13:56:03 pm
Όταν υπάρχει και θόρυβος στο σύστημα τότε έχουμε οπωσδήποτε ελεγκτή με δυναμική ανάδραση καταστάσεων?


Title: Re: [ΣΑΕ ΙΙ] Θέματα 2015
Post by: tsakos on September 14, 2015, 14:00:40 pm
Με μεγάλη επιφύλαξη επισυναπτω τη λύση του 4ου θέματος Ιούνιος 15. Αν δεν είναι σωστό πείτε προς αποφυγήν αυτού..
πως δειχνεις οτι p1<a/c p2<1/c

Να σου πω την αλήθεια δεν κατάλαβα που θα το χρησιμοποιήσω αυτό. Σίγουρα η λύση μου είναι ελλιπής. Επίσης όπου έχω r είναι δ ξεχάστηκα.


Title: Re: [ΣΑΕ ΙΙ] Θέματα 2015
Post by: c0ndemn3d on September 14, 2015, 14:25:06 pm
Με μεγάλη επιφύλαξη επισυναπτω τη λύση του 4ου θέματος Ιούνιος 15. Αν δεν είναι σωστό πείτε προς αποφυγήν αυτού..

Λάθος θέλει δυναμική ανάδραση, Routh και μετά χρησιμοποιείς τις ανισότητες που σου δίνει για να επιλέξεις τα k βάσει αυτών και όχι των α, β, γ.



Όταν υπάρχει και θόρυβος στο σύστημα τότε έχουμε οπωσδήποτε ελεγκτή με δυναμική ανάδραση καταστάσεων?

Όταν υπάρχει σταθερή διαταραχή κάνεις δυναμική, αλλιώς με κλασικό γραμμικό ελεγκτή έχεις σφάλμα.



Στο Θέμα 3 του ιουνίου 2015, πως αποδεικνύουμε την ολική ασυμπτωτική ευστάθεια?? Σε προηγούμενο post o c0ndemn3d λέει οτι αφού βρούμε οτι η παράγωγος της Lyapunov είναι αρνητικά ημιορισμένη μετά κάνουμε Lasalle. Με το Lasalle ομως βρίσκουμε τοπική ασυμπτωτική ευστάθεια ενώ εμείς θέλουμε ολική.. Ξέρει κανείς??

Εφόσον βρήκες ότι είναι μικρότερη ίση του 0 παντού, τότε η ασυμπτωτική ευστάθεια είναι ολική.


Title: Re: [ΣΑΕ ΙΙ] Θέματα 2015
Post by: Conflict on September 14, 2015, 14:26:51 pm
Με μεγάλη επιφύλαξη επισυναπτω τη λύση του 4ου θέματος Ιούνιος 15. Αν δεν είναι σωστό πείτε προς αποφυγήν αυτού..

Τα α και β ειναι μια χαρα. Τωρα για το γ ειχε αναφερθει σε προηγουμενο ποστ οτι χρειαζεται δυναμικη αναδραση καταστασεων αλλα εγω δεν καταφερα να την κανω.
Μπορει καποιος να επιβεβαιωσει ποια ειναι η σωστη λυση? :-\


Title: Re: [ΣΑΕ ΙΙ] Θέματα 2015
Post by: tsakos on September 14, 2015, 14:28:42 pm
Με μεγάλη επιφύλαξη επισυναπτω τη λύση του 4ου θέματος Ιούνιος 15. Αν δεν είναι σωστό πείτε προς αποφυγήν αυτού..

Λάθος θέλει δυναμική ανάδραση, Routh και μετά χρησιμοποιείς τις ανισότητες που σου δίνει για να επιλέξεις τα k βάσει αυτών και όχι των α, β, γ.

Ευχαριστώ..Μήπως ξέρεις και να πεις πότε χρησιμοποιείτε η δυναμική ανάδραση? Δηλαδή γιατί εδώ την χρησιμοποιούμε?


Title: Re: [ΣΑΕ ΙΙ] Θέματα 2015
Post by: Conflict on September 14, 2015, 14:29:13 pm
Με μεγάλη επιφύλαξη επισυναπτω τη λύση του 4ου θέματος Ιούνιος 15. Αν δεν είναι σωστό πείτε προς αποφυγήν αυτού..

Λάθος θέλει δυναμική ανάδραση, Routh και μετά χρησιμοποιείς τις ανισότητες που σου δίνει για να επιλέξεις τα k βάσει αυτών και όχι των α, β, γ.

Ειναι ευκολο να εξηγησεις λιγο πως θα γινει η δυναμικη αναδραση καταστασεων?
Καταρχην εφοσον το συστημα εχει γραμμικοποιηθει στο α ερωτημα εχει ως σημειο ισορροπιασ το (0,0) πλεον ή οχι?


Title: Re: [ΣΑΕ ΙΙ] Θέματα 2015
Post by: c0ndemn3d on September 14, 2015, 14:37:08 pm
Ειναι ευκολο να εξηγησεις λιγο πως θα γινει η δυναμικη αναδραση καταστασεων?
Καταρχην εφοσον το συστημα εχει γραμμικοποιηθει στο α ερωτημα εχει ως σημειο ισορροπιασ το (0,0) πλεον ή οχι?

Όταν έχεις σταθερή διαταραχή είναι σίγουρο ότι θα κάνεις δυναμική ανάδραση. Εδώ όμως κάνεις γιατί αν πας με γραμμική θα βγουν όλα συναρτήσει των α,β,γ τα οποία ΔΕΝ ξέρεις άρα ΔΕΝ μπορείς να βάλεις στον ελεγκτή σου. Με τη δυναμική ανάδραση ορίζοντας την τρίτη κατάσταση αναγκάζεις την έξοδο να γίνει ίση με το σήμα αναφοράς.



Με μεγάλη επιφύλαξη επισυναπτω τη λύση του 4ου θέματος Ιούνιος 15. Αν δεν είναι σωστό πείτε προς αποφυγήν αυτού..

Τα α και β ειναι μια χαρα. Τωρα για το γ ειχε αναφερθει σε προηγουμενο ποστ οτι χρειαζεται δυναμικη αναδραση καταστασεων αλλα εγω δεν καταφερα να την κανω.
Μπορει καποιος να επιβεβαιωσει ποια ειναι η σωστη λυση? :-\

Κάνεις δυναμική ανάδραση και μετά απλά θες ευστάθεια του ΧΠ. Χρησιμοποιείς τις 2 ανισότητες για να βρεις τα k, έτσι ώστε να μην έχουν α, β, γ τα οποία δεν είναι γνωστά (σε αυτά τα θέματα εξετάστηκα).


Title: Re: [ΣΑΕ ΙΙ] Θέματα 2015
Post by: nlogn on September 15, 2015, 00:29:40 am
Στο θέμα 4ο του Φεβρουαρίου του 2015, το α δεν ξέρω κατά πόσο λύνεται με Lyapunov...

Αν κάνεις αλλαγή συντεταγμένων σε πολικές θα βγάλεις:
   dr/dt = r * ( 4 - r^2 )
   από την παραπάνω διαφορική βλέπεις ότι θα πάει στον κύκλο με ακτίνα ρ= 2 και κέντρο το 0.

Για το β συνεχίζεις με κανονική lyapunov και ξεχνάς τις ταρζανιές του α.

Επίσης αυτή λέγεται εξίσωση επιμελητείας ... just saying...


Title: Re: [ΣΑΕ ΙΙ] Θέματα 2015
Post by: nikos1 on September 15, 2015, 18:06:56 pm
Αν μπορει καποιο καλο παιδι να πει πως ηταν τα Σ,Λ σεπτ 2015 θα ημουν ευγνωμων  :)


Title: Re: [ΣΑΕ ΙΙ] Θέματα 2015
Post by: Give_Life_Back on September 15, 2015, 18:14:24 pm
Αν μπορει καποιο καλο παιδι να πει πως ηταν τα Σ,Λ σημερα θα ημουν ευγνωμων  :)

Δεν ξέρω αν είναι σωστό αυτό που σου λέω εγώ τα έβαλα Σ,Σ,Λ,Λ,Σ . Με ένα παιδί που μίλησα μου είπε ότι το πρωτο είναι Λ. Τρέχα γύρευε


Title: Re: [ΣΑΕ ΙΙ] Θέματα 2015
Post by: c0ndemn3d on September 15, 2015, 18:16:07 pm
Μπορείτε να πείτε τι Σ-Λ είχε αν τα θυμάστε;


Title: Re: [ΣΑΕ ΙΙ] Θέματα 2015
Post by: Dealan on September 15, 2015, 20:29:45 pm
1) Ελεγξιμότητα σημαίνει ότι μπορείς από μια κατάσταση να οδηγήσεις το σύστημα σε μια άλλη σε πεπερασμένο χρόνο.

2) Η ελεγξιμότητα είναι ικανή και αναγκαία συνθήκη για να εφαρμόσεις γραμμική ανάδραση καταστάσεων.

(Στο περίπου και τα 2, άλλα δεν θυμάμαι.)


Title: Re: [ΣΑΕ ΙΙ] Θέματα 2015
Post by: giwrgosbg on September 16, 2015, 00:00:58 am
Αν μπορει καποιο καλο παιδι να πει πως ηταν τα Σ,Λ σημερα θα ημουν ευγνωμων  :)

Δεν ξέρω αν είναι σωστό αυτό που σου λέω εγώ τα έβαλα Σ,Σ,Λ,Λ,Σ . Με ένα παιδί που μίλησα μου είπε ότι το πρωτο είναι Λ. Τρέχα γύρευε
Μηπως θυμάστε τι ελέγε το δευτερο ειναι το μονο που δεν θυμάμαι με τίποτα


Title: Re: [ΣΑΕ ΙΙ] Θέματα 2015
Post by: orestisf on October 21, 2015, 13:24:45 pm
Μπορείτε να πείτε τι Σ-Λ είχε αν τα θυμάστε;

ολα Σ, ειδα το γραπτο σμρ


Title: Re: [ΣΑΕ ΙΙ] Θέματα 2015
Post by: PanosGo on February 13, 2016, 20:13:11 pm
Καλησπέρα!
Σεπτέμβριος 15 Πρώτο θέμα... Ξέρει κανείς τί κάνουμε για την ευστάθεια;;
Με υποψήφια Lyapunov την V=1/2*x1^2 + 1/2*x2^2 η παράγωγός της βγάζει έναν "αμφισβητούμενης αρνητικότητας" όρο (x1^2*x2-x1^2*x2*γ)


Title: Re: [ΣΑΕ ΙΙ] Θέματα 2015
Post by: Exomag on February 13, 2016, 20:19:12 pm
Καλησπέρα!
Σεπτέμβριος 15 Πρώτο θέμα... Ξέρει κανείς τί κάνουμε για την ευστάθεια;;
Με υποψήφια Lyapunov την V=1/2*x1^2 + 1/2*x2^2 η παράγωγός της βγάζει έναν "αμφισβητούμενης αρνητικότητας" όρο (x1^2*x2-x1^2*x2*γ)

Πρέπει να ορίσεις την Lyapunov λίγο διαφορετικά, ώστε να εξαφανίσεις (στην παράγωγο της Lyapunov) κάποια/ες σταθερά/ες που σε ενοχλεί/ούν και έτσι να οδηγηθείς σε απαλοιφές.


Title: Re: [ΣΑΕ ΙΙ] Θέματα 2015
Post by: c0ndemn3d on February 13, 2016, 20:20:54 pm
Δεν έχω δει το θέμα, αλλά μάλλον η Lyapunov θα είναι της μορφής V = 1/(2γ) x_1^2 + 1/2 x_2^2 ή V = 1/2 x_1^2 + 1/(2γ) x_2^2


Title: Re: [ΣΑΕ ΙΙ] Θέματα 2015
Post by: PanosGo on February 13, 2016, 20:32:41 pm
Είστε τρελά παλικάρια!! Ευχαριστώ πολύ!!


Title: Re: [ΣΑΕ ΙΙ] Θέματα 2015
Post by: Παναγιώτης on February 13, 2016, 20:55:11 pm
Στο ίδιο θέμα (Σεπτ 2015, θέμα πρώτο) στο γ που ζητάει την ευρωστία τι κάνουμε? Γιατί στις σημειώσεις δεν βρήκα κάτι εκτός από το τέλους που έχει κάτι πίνακες. Μήπως εννοεί την αμεταβλητότητα?


Title: Re: [ΣΑΕ ΙΙ] Θέματα 2015
Post by: Exomag on February 13, 2016, 22:46:09 pm
Στο ίδιο θέμα (Σεπτ 2015, θέμα πρώτο) στο γ που ζητάει την ευρωστία τι κάνουμε? Γιατί στις σημειώσεις δεν βρήκα κάτι εκτός από το τέλους που έχει κάτι πίνακες. Μήπως εννοεί την αμεταβλητότητα?

Θέλει να κάνεις πάλι ανάλυση Lyapunov, με σκοπό να δεις κατά πόσο το σύστημα είναι ασταθές ή όχι (και αν όχι, πως συμπεριφέρεται) παρουσία της σταθερής διαταραχής που λέει.

Checkαρε το τελευταίο κομμάτι των σημειώσεων (https://www.thmmy.gr/smf/index.php?action=tpmod;dl=item2671) και των παραδειγμάτων (https://www.thmmy.gr/smf/index.php?action=tpmod;dl=item2810), που είναι σχετικά με αυτό που ζητάει (ανάλυση ευρωστίας, δηλαδή).


Title: Re: [ΣΑΕ ΙΙ] Θέματα 2015
Post by: Laharl on February 14, 2016, 21:24:30 pm
Kαμια ιδέα για Σεπτέμβρη 2015 3ο θέμα??


Title: Re: [ΣΑΕ ΙΙ] Θέματα 2015
Post by: Exomag on February 14, 2016, 22:03:02 pm
Kαμια ιδέα για Σεπτέμβρη 2015 3ο θέμα??

Ερώτημα α
Εφαρμόζεις γραμμικη ανάδραση καταστάσεων, βρίσκεις τη συνάρτηση μεταφοράς του συστήματος κλειστού βρόχου που θα προκύψει, βρίσκεις σχέση που να συνδέει τη φυσική συχνότητα και το συντελεστή απόσβεσης με τα κέρδη της γραμμικής ανάδρασης, και λύνεις ως προς τα κέρδη.


Title: Re: [ΣΑΕ ΙΙ] Θέματα 2015
Post by: PanosGo on February 15, 2016, 16:48:51 pm
Τώρα που πήραμε φόρα, καμιά ιδέα για του Ιούνη του 15 το 4γ;;
Με τις ανισότητες τί παίζει;; Έκανα το σύστημα σε ελέγξιμη κανονική μορφή, αλλά δεν μου εμφανίστηκαν πουθενά...


Title: Re: [ΣΑΕ ΙΙ] Θέματα 2015
Post by: c0ndemn3d on February 15, 2016, 17:00:11 pm
Τι ελεγκτή παίρνεις; Πώς δικαιολογείς την ευστάθεια του συστήματος; Στον ελεγκτή χρησιμοποιείς άγνωστες ποσότητες; Είναι ψιλοδύσκολο ερώτημα να το κάνεις 100% σωστά αυτό


Title: Re: [ΣΑΕ ΙΙ] Θέματα 2015
Post by: Laharl on February 15, 2016, 19:21:51 pm
Kαμια ιδέα για Σεπτέμβρη 2015 3ο θέμα??

Ερώτημα α
Εφαρμόζεις γραμμικη ανάδραση καταστάσεων, βρίσκεις τη συνάρτηση μεταφοράς του συστήματος κλειστού βρόχου που θα προκύψει, βρίσκεις σχέση που να συνδέει τη φυσική συχνότητα και το συντελεστή απόσβεσης με τα κέρδη της γραμμικής ανάδρασης, και λύνεις ως προς τα κέρδη.

Αυτή την σχέση που να συνδέει φυσική συχνότητα και συντελεστή απόσβεσης με τα κέρδη δεν μπορώ να καταλάβω ποια είναι. Μπορείς να το κάνεις λίγο πιο λιανά? Θενκς ;)


Title: Re: [ΣΑΕ ΙΙ] Θέματα 2015
Post by: Exomag on February 15, 2016, 19:38:43 pm
Kαμια ιδέα για Σεπτέμβρη 2015 3ο θέμα??

Ερώτημα α
Εφαρμόζεις γραμμικη ανάδραση καταστάσεων, βρίσκεις τη συνάρτηση μεταφοράς του συστήματος κλειστού βρόχου που θα προκύψει, βρίσκεις σχέση που να συνδέει τη φυσική συχνότητα και το συντελεστή απόσβεσης με τα κέρδη της γραμμικής ανάδρασης, και λύνεις ως προς τα κέρδη.

Αυτή την σχέση που να συνδέει φυσική συχνότητα και συντελεστή απόσβεσης με τα κέρδη δεν μπορώ να καταλάβω ποια είναι. Μπορείς να το κάνεις λίγο πιο λιανά? Θενκς ;)

Αφού κάνεις τη γραμμική ανάδραση, θα βρεις το χαρακτηριστικό πολυώνυμο του συστήματος κλειστού βρόχου που δημιούργησες.

Αυτό θα είναι της μορφής s2 + as + b = 0, και από τη θεωρία ΣΑΕ Ι ξέρουμε πως a = 2*ζ*ωn και b = ωn2.


Title: Re: [ΣΑΕ ΙΙ] Θέματα 2015
Post by: Laharl on February 15, 2016, 22:12:15 pm
Kαμια ιδέα για Σεπτέμβρη 2015 3ο θέμα??

Ερώτημα α
Εφαρμόζεις γραμμικη ανάδραση καταστάσεων, βρίσκεις τη συνάρτηση μεταφοράς του συστήματος κλειστού βρόχου που θα προκύψει, βρίσκεις σχέση που να συνδέει τη φυσική συχνότητα και το συντελεστή απόσβεσης με τα κέρδη της γραμμικής ανάδρασης, και λύνεις ως προς τα κέρδη.

Αυτή την σχέση που να συνδέει φυσική συχνότητα και συντελεστή απόσβεσης με τα κέρδη δεν μπορώ να καταλάβω ποια είναι. Μπορείς να το κάνεις λίγο πιο λιανά? Θενκς ;)

Αφού κάνεις τη γραμμική ανάδραση, θα βρεις το χαρακτηριστικό πολυώνυμο του συστήματος κλειστού βρόχου που δημιούργησες.

Αυτό θα είναι της μορφής s2 + as + b = 0, και από τη θεωρία ΣΑΕ Ι ξέρουμε πως a = 2*ζ*ωn και b = ωn2.

Σόρυ που σε πρήζω αλλά θέλω να δω εάν το κατάλαβα σωστά...

Θα βρούμε μέσω αυτού του συστήματος ένα a= λκ2+ γκ1  και b=λκ1 (εάν κάνω σωστά πράξεις) και kr=b

Άρα θα έχω a=2ζω συνεπαγεται λκ2+γκ1=2ζω και b=ω^2 συνεπαγεται λκ1=ω^2 οπου ω=2 και ζ=ριζα2/2

Άρα όλα μου τα κ1,κ2 ειναι συναρτηση του λ,γ και το κr = b

Σωστά?


Title: Re: [ΣΑΕ ΙΙ] Θέματα 2015
Post by: El Niño on February 15, 2016, 23:23:47 pm
Στο ίδιο θέμα (Σεπτ 2015, θέμα πρώτο) στο γ που ζητάει την ευρωστία τι κάνουμε? Γιατί στις σημειώσεις δεν βρήκα κάτι εκτός από το τέλους που έχει κάτι πίνακες. Μήπως εννοεί την αμεταβλητότητα?

Θέλει να κάνεις πάλι ανάλυση Lyapunov, με σκοπό να δεις κατά πόσο το σύστημα είναι ασταθές ή όχι (και αν όχι, πως συμπεριφέρεται) παρουσία της σταθερής διαταραχής που λέει.

Checkαρε το τελευταίο κομμάτι των σημειώσεων (https://www.thmmy.gr/smf/index.php?action=tpmod;dl=item2671) και των παραδειγμάτων (https://www.thmmy.gr/smf/index.php?action=tpmod;dl=item2810), που είναι σχετικά με αυτό που ζητάει (ανάλυση ευρωστίας, δηλαδή).

ουτε γω καταλαβα ακριβως ποια τα βηματα που πρεπει να ακολουθησουμε.. αν μπορεις να το εξηγησεις καπως..


Title: Re: [ΣΑΕ ΙΙ] Θέματα 2015
Post by: c0ndemn3d on February 15, 2016, 23:43:29 pm
Πρέπει να δεις απλώς αν το σύστημα οδηγείται σε αστάθεια ε με Lyapunov ας πούμε και μετά να κάνεις κάποια σχόλια για το σφάλμα μόνιμης κατάστασης (αν ειναι ελεγχόμενο κτλπ)


Title: Re: [ΣΑΕ ΙΙ] Θέματα 2015
Post by: El Niño on February 16, 2016, 00:21:38 am
Πρέπει να δεις απλώς αν το σύστημα οδηγείται σε αστάθεια ε με Lyapunov ας πούμε και μετά να κάνεις κάποια σχόλια για το σφάλμα μόνιμης κατάστασης (αν ειναι ελεγχόμενο κτλπ)

πφφ.. δεν νομιζω οτι το χω.. αλλα θενξ..

θεμα 3 του 15, παρολο που ο exomag το εξηγει παραπανω , δν καταλαβαινω πως πρεπει να αρχισω...

πρεπει να βρω τον πινακα Α-ΒΚ ?


ακυρο... αποτι καταλαβα τελικως βρισκω SI-A+BK παιρνω το χαρακτηριστικο και εξισωνω οπως λεει ο Laharl προηγουμενως


Title: Re: [ΣΑΕ ΙΙ] Θέματα 2015
Post by: El Niño on February 16, 2016, 00:27:36 am
Σωστά?
σε αυτο το συμπερασμα εφτασα κ γω... ασχετα που δεν ειχα ιδεα για τον τυπο s^2+ 2ζωn + ωn^2 :P


για ερωτημα β και γ δεν ειμαι σιγουρος καθολου τι θελει

παιζει να θελει να βρω τις ιδιοτιμες απο το χ.π. και μετα να κρατησω αυτη με το μικροτερο αρνητικο μερος, μηδενιζοντας το Κ που αντιστοιχει στη 2η ? (κ2=0 και βρισκω σχεση λ,γ...)


Title: Re: [ΣΑΕ ΙΙ] Θέματα 2015
Post by: El Niño on February 16, 2016, 01:00:09 am
Πρέπει να δεις απλώς αν το σύστημα οδηγείται σε αστάθεια ε με Lyapunov ας πούμε και μετά να κάνεις κάποια σχόλια για το σφάλμα μόνιμης κατάστασης (αν ειναι ελεγχόμενο κτλπ)

παιρνοντας ιδια lyapunov με το α ερωτημα , κατεληξα οτι V' = -x1^2 + x1V0... πρεπει να διωξω με καποια αλλη lyapunov το V0 η κατι αλλο?


σορρυ για το σπαμ γενικα, ρωταω και οποιος μπορει απαντα....

ΣΑΕ 2 το τελευταιο μεγαλο εμποδιο για πτυχιο  :D :D


Title: Re: [ΣΑΕ ΙΙ] Θέματα 2015
Post by: Laharl on February 16, 2016, 14:26:21 pm
Σωστά?
σε αυτο το συμπερασμα εφτασα κ γω... ασχετα που δεν ειχα ιδεα για τον τυπο s^2+ 2ζωn + ωn^2 :P



To συμπέρασμα εν τέλη όμως ποιό είναι? Απλά του λέμε πάρε κ1,κ2 συναρτήση των γ,λ ?


Title: Re: [ΣΑΕ ΙΙ] Θέματα 2015
Post by: di_em on February 16, 2016, 14:59:08 pm
Ναι, τα γ, λ τα θεωρεί γνωστά λογικά για να τα δίνει...


Title: Re: [ΣΑΕ ΙΙ] Θέματα 2015
Post by: El Niño on February 16, 2016, 20:13:09 pm
οταν ζηταει να δειξουμε οτι το 0,0 ειναι μοναδικο ΣΙ , πρεπει να κανουμε και το θεωρημα του Lasalle ?


Title: Re: [ΣΑΕ ΙΙ] Θέματα 2015
Post by: status_quo on February 16, 2016, 20:16:15 pm
νομίζω πως όχι. Τουλάχιστον έτσι φαίνεται κι από τα παραδείγματα που είναι ανεβασμένα από το μάθημα


Title: Re: [ΣΑΕ ΙΙ] Θέματα 2015
Post by: El Niño on February 16, 2016, 20:26:08 pm
απλα σελιδα 18 απο τις σημειωσεις του exomag πχ το χρησιμοποιει για να δειξει οτι το 0,0 ειναι το μοναδικο σημειο ισορροπιας...

απο την αλλη.. στο θεμα 1 του σεπτ 15 , δεν μου φαινεται οτι το χρειαζεται....
ομως, μου φαινεται πολυ απλο να λυσω απλα μια εξισωση για 1 μοναδα..


Title: Re: [ΣΑΕ ΙΙ] Θέματα 2015
Post by: Jeik on February 16, 2016, 20:36:29 pm
Το Lasalle χρειάζεται μόνo αν η παράγωγος της V είναι <=0 (αντί για <0).


Title: Re: [ΣΑΕ ΙΙ] Θέματα 2015
Post by: Andromedas on February 16, 2016, 20:36:59 pm
οταν ζηταει να δειξουμε οτι το 0,0 ειναι μοναδικο ΣΙ , πρεπει να κανουμε και το θεωρημα του Lasalle ?
Γενικά Σημεία ισορροπίας είναι όταν x'=0. Και εάν είναι γραμμικό υπάρχει μόνο ένα στο σύστημα.

Όταν σου πει με lyapunov δες ευστάθεια του συστήματος και βγάλεις πχ την  V'(x)=0 για x=(0,R)
δηλαδή δεν υπάρχει ο όρος x2 και δεν μηδενίζεται μόνο για V'(0)=0
πχ V'(x)=-x1^2 τότε για V'(x)=0 έχω x=(0,R) και κάνω Lassal

Αν βγαίνει V'(x)=-x1^2 (x2 -5)^2 τότε έχω για V(x)=0 x=(0,R) ,(R,5) και πρέπει να κάνω Lassal


Title: Re: [ΣΑΕ ΙΙ] Θέματα 2015
Post by: El Niño on February 16, 2016, 20:42:21 pm
οκ παιδια το χω νομιζω ... ευχαριστω :)


Title: Re: [ΣΑΕ ΙΙ] Θέματα 2015
Post by: JakeLaMotta on February 17, 2016, 01:20:23 am
Έχει λύσει κανείς το θέμα 1γ) του Σεπτεμβρίου του 2015,που ρωτάει αν το σύστημα είναι εύρωστο??Γιατί δεν καταλαβαίνω και πολλά από την εξήγηση που δίνουν κάποιοι στη προηγούμενη σελίδα...


Title: Re: [ΣΑΕ ΙΙ] Θέματα 2015
Post by: El Niño on February 17, 2016, 01:24:24 am
Έχει λύσει κανείς το θέμα 1γ) του Σεπτεμβρίου του 2015,που ρωτάει αν το σύστημα είναι εύρωστο??Γιατί δεν καταλαβαίνω και πολλά από την εξήγηση που δίνουν κάποιοι στη προηγούμενη σελίδα...

εγω κανοντας απλα lyapunov παλι, με την ιδια συναρτηση απλα βαζοντας τωρα και το v0 κατεληξα σε ενα V'= -x1^2 + x1V0.. και δεν ξερω πως να συνεχισω μετα


Title: Re: [ΣΑΕ ΙΙ] Θέματα 2015
Post by: JakeLaMotta on February 17, 2016, 01:33:00 am
Έχει λύσει κανείς το θέμα 1γ) του Σεπτεμβρίου του 2015,που ρωτάει αν το σύστημα είναι εύρωστο??Γιατί δεν καταλαβαίνω και πολλά από την εξήγηση που δίνουν κάποιοι στη προηγούμενη σελίδα...

εγω κανοντας απλα lyapunov παλι, με την ιδια συναρτηση απλα βαζοντας τωρα και το v0 κατεληξα σε ενα V'= -x1^2 + x1V0.. και δεν ξερω πως να συνεχισω μετα

Ναι,αυτό κι εγώ το έκανα,αλλά δε βλέπω να οδηγεί κάπου... :P
Σίγουρα κάπως θα πρέπει να αξιοποιούμε και την ανισότητα r0>v0,δεν μπορεί να το δίνει χωρίς λόγο...


Title: Re: [ΣΑΕ ΙΙ] Θέματα 2015
Post by: c0ndemn3d on February 17, 2016, 01:37:23 am
Πρέπει να πάρετε τα μέτρα (δείτε λυμένες ασκήσεις - δεν έχω δει το θέμα)


Title: Re: [ΣΑΕ ΙΙ] Θέματα 2015
Post by: JakeLaMotta on February 17, 2016, 01:41:38 am
Πρέπει να πάρετε τα μέτρα (δείτε λυμένες ασκήσεις - δεν έχω δει το θέμα)

Δεν βλέπω καμιά λυμένη άσκηση παρόμοια με αυτήν....είναι διαφορετικές περιπτώσεις εκεί


Title: Re: [ΣΑΕ ΙΙ] Θέματα 2015
Post by: PanteGrv on April 20, 2016, 12:22:28 pm
Εεεε... σίγουρα; Εγώ το μόνο που βλέπω στην παρατήρηση εκείνη είναι ενα σχόλιο για την ελεγξιμότητα... Γενικά αν υπήρχε λυμένη αυτή η άσκηση απο ροβιθάκη, μάλλον θα ήταν σε άλλο κεφάλαιο, αυτό το θέμα δεν έχει σχέση με ελεγξιμότητα ( νομίζω δηλαδή  :P )

Edit: Τέλως πάντων, εγω θα ποσταρω αυτο που σκεφτηκα. Εμενα σωστο μου φαινεται, αλλα εγω ειμαι χαζος γενικα, οποτε πειτε και σεις...

Δινεται α>0 και β<0. Επισης ξερουμε οτι η συναρτηση lyapunov μπορει να ειναι οποιαδηποτε αρκει να ειναι θετικα ημιορισμενη σωστα; Επιλεγω την εξης:

V = 1/2α * χ12 - 1/2β * χ22 , η οποια ειναι θετικα ημιορισμενη διοτι β<0 και μαζι με το - που βαλαμε ειναι οκ...

Υπολογιζουμε την παραγωγο και μετα απο πραξεις εγω καταληγω σε αυτο:

 .
V = 1/αβ * ( βχ12 - αχ22 ) * ( 4 - χ12 - χ22 )

  • Το 1/αβ ειναι <0 παντα και παντου
  • Το βχ12 - αχ22 ειναι συνολικα <=0 ( =0 στο (0,0) ) γιατι αυτο ισχυει για καθε ορο, ειναι σαν αθροισμα τετραγωνων με ενα - απεξω
  • Ο τελευταιος ορος ειναι που αλλαζει προσημο αναλογα με το αν βρισκεται μεσα η εξω απο τον κυκλο

Οποτε ουσιαστικα εχουμε τις εξης περιπτωσεις

  • χ12 + χ22 < 4 . Τοτε η παραγωγος ειναι (-) * (-) * (+) = (+) οποτε οι καταστασεις θα κινουνται προς τον κυκλο μεχρι να τον φτασουν. Οταν φτασουν θα κινουνται πανω του.
  • χ12 + χ22 > 4 . Τοτε η παραγωγος ειναι (-) * (-) * (-) = (-) οποτε οι καταστασεις ξανα θα κινουνται προς τον κυκλο και οταν φτασουν θα κινουνται πανω του.

Με βαση αυτα καταληγω οτι ανεξαρτητα απο σημειο εκκινησης οι καταστασεις τελικα θα κινουνται πανω στον κυκλο με κεντρο (0,0) και ακτινα 2. Τι λετε;

α και β δεν είναι σταθερά. Ισχύει μάλιστα α=α(x) και β=β(x). Οπότε η παραγώγιση αυτής της lyapunov είναι λάθος γιατί δεν συμπεριλαμβάνει μερικές παραγώγους ως προς αυτά. Αντ αυτού επιλέγουμε την ίδια lyapunov με το α) ερώτημα (V=xT*I*x) και βγαίνει ένας παραπάνω όρος στην παράγωγό της, που με βάση αυτά που λέει για το α, β, x1, και x2 βγαίνει πάντα αρνητικός. Άρα τελικά βγαίνει το ζητούμενο (φραγμένη κίνηση).



Title: Re: [ΣΑΕ ΙΙ] Θέματα 2015
Post by: nvog1993 on June 02, 2016, 15:40:50 pm
Για το 3γ) του Σεπτ 15', είναι σωστό να πούμε ότι όσο τείνει το λ στο 0, τόσο τα κέρδη κ1 και κ2 τείνουν στο άπειρο και έτσι δυσκολεύει ο έλεγχος του συστήματος?


Title: Re: [ΣΑΕ ΙΙ] Θέματα 2015
Post by: prodrom on June 04, 2016, 22:38:51 pm
Μπορεί κάποιος να γράψει μια πιο αναλυτική περιγραφή για τη λύση που πρότεινε ο condemned στο θέμα 4γ του Ιουνίου του 15;


Title: Re: [ΣΑΕ ΙΙ] Θέματα 2015
Post by: nvog1993 on June 04, 2016, 23:30:30 pm
Μπορεί κάποιος να γράψει μια πιο αναλυτική περιγραφή για τη λύση που πρότεινε ο condemned στο θέμα 4γ του Ιουνίου του 15;
Εγώ έτσι θα το έκανα.


Title: Re: [ΣΑΕ ΙΙ] Θέματα 2015
Post by: AnnaG on June 04, 2016, 23:32:44 pm
Μπορεί κάποιος να γράψει μια πιο αναλυτική περιγραφή για τη λύση που πρότεινε ο condemned στο θέμα 4γ του Ιουνίου του 15;



Title: Re: [ΣΑΕ ΙΙ] Θέματα 2015
Post by: Μεταλλαγμένη Πάπια on June 05, 2016, 13:18:14 pm
α και β δεν είναι σταθερά. Ισχύει μάλιστα α=α(x) και β=β(x). Οπότε η παραγώγιση αυτής της lyapunov είναι λάθος γιατί δεν συμπεριλαμβάνει μερικές παραγώγους ως προς αυτά. Αντ αυτού επιλέγουμε την ίδια lyapunov με το α) ερώτημα (V=xT*I*x) και βγαίνει ένας παραπάνω όρος στην παράγωγό της, που με βάση αυτά που λέει για το α, β, x1, και x2 βγαίνει πάντα αρνητικός. Άρα τελικά βγαίνει το ζητούμενο (φραγμένη κίνηση).

Μπορείς να το αναλύσεις λίγο παραπάνω αυτο; Δεν λεει ρητα πουθενα οτι και τα δυο α,β είναι συναρτήσεις του χ, μονο οτι η σχεση μεταξυ τους αλλαζει για διαφορες τιμες του χ. (Ποιος λεει οτι δεν ισχυει α=σταθερο και β=β(χ) η το αντιθετο;). Ακομα και αν ισχυει οτι και τα δυο ειναι συναρτησεις του χ, δεν λεει ποια ακριβως ειναι αυτη, πως ακριβως θα τα βαλουμε στην παραγωγο;


Title: Re: [ΣΑΕ ΙΙ] Θέματα 2015
Post by: El Niño on June 05, 2016, 17:11:16 pm
ερωτηση

οταν εχω το κλασικα συστημα χ΄= Αx+Bu
εστω οτι πχ λεει γραμμικη αναδραση.. βαζω τον u= -k1x1-k2x2

μετα, για να τον περασω στον Α πως το παω?
πχ ας πουμε οτι ο Α ειναι  1 0 και ο Β= 1
                                       0 1              1

ο νεος Α θα παει  1-κ1  -κ2
                           -κ1    1-κ2   
???




Title: Re: [ΣΑΕ ΙΙ] Θέματα 2015
Post by: Exomag on June 05, 2016, 17:16:36 pm
ερωτηση

οταν εχω το κλασικα συστημα χ΄= Αx+Bu
εστω οτι πχ λεει γραμμικη αναδραση.. βαζω τον u= -k1x1-k2x2

μετα, για να τον περασω στον Α πως το παω?
πχ ας πουμε οτι ο Α ειναι  1 0 και ο Β= 1
                                       0 1              1

ο νεος Α θα παει  1-κ1  -κ2
                           -κ1    1-κ2   
???




Απλές πράξεις πινάκων είναι. Στο παράδειγμα που έγραψες, σωστά έγραψες τον "νέο Α".


Title: Re: [ΣΑΕ ΙΙ] Θέματα 2015
Post by: El Niño on June 05, 2016, 17:17:25 pm
ναι αρα καλα το εγραψα ετσι?
θελω να πω, το u παει και στις 2 σειρες του πινακα ε? οχι μονο στη 1 πχ την πανω?


Title: Re: [ΣΑΕ ΙΙ] Θέματα 2015
Post by: Exomag on June 05, 2016, 17:20:03 pm
ναι αρα καλα το εγραψα ετσι?
θελω να πω, το u παει και στις 2 σειρες του πινακα ε? οχι μονο στη 1 πχ την πανω?

Καλά το έγραψες, ναι.

Όταν βάζεις γραμμικη ανάδραση καταστάσεων u = - k1x1 - k2x2 = - kx, όπου x το Nx1 διάνυσμα καταστάσεων και k το 1xN διάνυσμα των θετικών κερδών, ο "νέος Α" θα είναι ίσος με τον NxN πίνακα A - BK.


Title: Re: [ΣΑΕ ΙΙ] Θέματα 2015
Post by: El Niño on June 05, 2016, 17:22:28 pm
ωραια θενξ,
αναφερομουν συγκεκριμενα θεμα 3 σεπτ 15 α ερωτημα


Title: Re: [ΣΑΕ ΙΙ] Θέματα 2015
Post by: Exomag on June 05, 2016, 17:38:38 pm
ωραια θενξ,
αναφερομουν συγκεκριμενα θεμα 3 σεπτ 15 α ερωτημα

Οκ. Να ξέρεις πως ο τύπος που έγραψα προφανώς ισχύει σε όλα τα γραμμικά συστήματα όπου χρησιμοποιείται γραμμική ανάδραση καταστάσεων, οπότε μπορείς να τον χρησιμοποιείς σε οποιαδήποτε τέτοια περίπτωση.


Title: Re: [ΣΑΕ ΙΙ] Θέματα 2015
Post by: El Niño on June 05, 2016, 17:42:26 pm
Οκ. Να ξέρεις πως ο τύπος που έγραψα προφανώς ισχύει σε όλα τα γραμμικά συστήματα όπου χρησιμοποιείται γραμμική ανάδραση καταστάσεων, οπότε μπορείς να τον χρησιμοποιείς σε οποιαδήποτε τέτοια περίπτωση.
ναι ναι, αυτο ειχα στο μυαλο μου,
απλα στην επιλυση θεματων προεκυψε διαφωνια, και ειπα να το σιγουρεψω,

θενξ :)


Title: Re: [ΣΑΕ ΙΙ] Θέματα 2015
Post by: El Niño on June 05, 2016, 17:44:45 pm
για ιδιο θεμα (σεπτ 15/3)

β) κ2= 0 καθως δεν επηραζει τα ζ και ω που δινονται και βρισκω σχεση για γ-λ ?

γ) παιρνω Μ, για λ=0 detM = 0 αρα δεν ειναι ελεγξιμο και τελος?


Title: Re: [ΣΑΕ ΙΙ] Θέματα 2015
Post by: nvog1993 on June 05, 2016, 18:24:47 pm
για ιδιο θεμα (σεπτ 15/3)
γ) παιρνω Μ, για λ=0 detM = 0 αρα δεν ειναι ελεγξιμο και τελος?
Ίσως, επειδή λέει στη γειτονία του 0, και όχι 0, που όντως θα ήταν μη ελέγξιμο, να πρέπει να πεις ότι τα κέρδη τείνουν στο άπειρο και έτσι δυσκολεύει πάρα πολύ η διαδικασία του ελέγχου


Title: Re: [ΣΑΕ ΙΙ] Θέματα 2015
Post by: prodrom on June 05, 2016, 19:46:13 pm
Μπορεί κάποιος να περιγράψει την ανάλυση του για το θέμα 1γ του Σεπτεμβρίου του 2015, αφού βρει τη παράγωγο της Lyapunov και μετά;


Title: Re: [ΣΑΕ ΙΙ] Θέματα 2015
Post by: nvog1993 on June 05, 2016, 21:06:20 pm
Μπορεί κάποιος να περιγράψει την ανάλυση του για το θέμα 1γ του Σεπτεμβρίου του 2015, αφού βρει τη παράγωγο της Lyapunov και μετά;
Εγώ βρήκα το νέο ΣΙ αφού v(t) = v0 σταθερό, το οποίο είναι το χ* (0,v0/r0) . Με αλλαγή μεταβλητής, lyapunov και lasalle βγάζω το
z* (0,0) και άρα το x* (0,v0/r0) ασυμπτωτικά ευσταθές. Και αφού το v0 αρκούντως μικρό, με το v0/r0 ακόμα πιο μικρό, μπορώ να πω ότι το σύστημά μου είναι εύρωστο αφού θα ισορροπήσει σε ένα σημείο αρκετά κοντά στο αρχικό (0,0).


Title: Re: [ΣΑΕ ΙΙ] Θέματα 2015
Post by: El Niño on June 06, 2016, 01:14:01 am
Εγώ βρήκα το νέο ΣΙ αφού v(t) = v0 σταθερό, το οποίο είναι το χ* (0,v0/r0) . Με αλλαγή μεταβλητής, lyapunov και lasalle βγάζω το
z* (0,0) και άρα το x* (0,v0/r0) ασυμπτωτικά ευσταθές. Και αφού το v0 αρκούντως μικρό, με το v0/r0 ακόμα πιο μικρό, μπορώ να πω ότι το σύστημά μου είναι εύρωστο αφού θα ισορροπήσει σε ένα σημείο αρκετά κοντά στο αρχικό (0,0).

αν κανεις αλλαγη μεταβλητης, με z1=x1 δλδ και z2=x2-v0/r0 και βρεις μετα τον Α~

δεν μπορεις απλα να βρεις ιδιοτιμες αυτου? και αν ειναι αρνητικες να πεις ολα καλα?

ακυρο, μαλλον αφου δεν ειναι γραμμικο συστημα, φανταζομαι κανεις οτι σελιδα 15 στις σημειωσεις


Title: Re: [ΣΑΕ ΙΙ] Θέματα 2015
Post by: PanteGrv on June 06, 2016, 21:49:33 pm
Μπορείς να το αναλύσεις λίγο παραπάνω αυτο; Δεν λεει ρητα πουθενα οτι και τα δυο α,β είναι συναρτήσεις του χ, μονο οτι η σχεση μεταξυ τους αλλαζει για διαφορες τιμες του χ. (Ποιος λεει οτι δεν ισχυει α=σταθερο και β=β(χ) η το αντιθετο;). Ακομα και αν ισχυει οτι και τα δυο ειναι συναρτησεις του χ, δεν λεει ποια ακριβως ειναι αυτη, πως ακριβως θα τα βαλουμε στην παραγωγο;

δεν ξέρεις ακριβώς τι παίζει με αυτή την σχέση, όμως δεν παύει ανάλογα με την κατάσταση να αλλάζει η σχέση μεταξύ τους όπως λες. άρα μπορεί να μην είναι καν συνεχείς οι συναρτήσεις  α(x) και β(x). ακριβώς επειδή δεν ξέρεις τι σχέση είναι αυτή, αλλά υποπτεύεσαι ότι υπάρχει, δεν μπορείς να τις εισάγεις καθόλου στην συνάρτηση lyapunov (οι παράγωγοί τους σου είναι άγνωστες). αν πας με την lyapunov του α ερωτήματος βγαίνει τελικά φραγμένη κίνηση. Δεν βγαίνει όμως ότι θα πάει στον κύκλο που πήγαινε πριν, γι αυτό και η γενικότερη απαίτηση "φραγμένη κίνηση"


Title: Re: [ΣΑΕ ΙΙ] Θέματα 2015
Post by: Cern O' Bill on June 07, 2016, 10:07:45 am
Εγώ έτσι θα το έκανα.

Εδώ δεν πρέπει να πάρουμε γραμμικό ελεγκτή?? Δηλαδή μορφή u=-Kx+Kr*r και όχι -Κx-Ki*z που είναι δυναμικής ανάδρασης?


Title: Re: [ΣΑΕ ΙΙ] Θέματα 2015
Post by: nvog1993 on June 07, 2016, 10:37:22 am
Εδώ δεν πρέπει να πάρουμε γραμμικό ελεγκτή?? Δηλαδή μορφή u=-Kx+Kr*r και όχι -Κx-Ki*z που είναι δυναμικής ανάδρασης?
Εδώ δεν έχεις ακριβή γνώση των παραμέτρων του συστήματος (α,b,c) και γι αυτό προτιμάται η δυναμική ανάδραση που δεν απαιτεί αυτή τη γνώση.


Title: Re: [ΣΑΕ ΙΙ] Θέματα 2015
Post by: Cern O' Bill on June 07, 2016, 10:53:37 am
Εδώ δεν έχεις ακριβή γνώση των παραμέτρων του συστήματος (α,b,c) και γι αυτό προτιμάται η δυναμική ανάδραση που δεν απαιτεί αυτή τη γνώση.

Η εκφώνηση όμως δε ζητάει γραμμικό ελεγκτή ανάδρασης καταστάσεων?? Έχουμε αυτήν τη δυνατότητα?


Title: Re: [ΣΑΕ ΙΙ] Θέματα 2015
Post by: nvog1993 on June 07, 2016, 10:59:05 am
Η εκφώνηση όμως δε ζητάει γραμμικό ελεγκτή ανάδρασης καταστάσεων?? Έχουμε αυτήν τη δυνατότητα?
Όταν λέμε γραμμικό ελεγκτή, θέλουμε απλά να είναι στην μορφή u = -k1x1 -k2x2 -k*(z ή r). Δηλαδή, απλά να έχεις μια είσοδο συναρτήσει των καταστάσεων του συστήματος που να είναι γραμμική. Στην δυναμική ανάδραση προσθέτεις ακόμα μια μεταβλητή κατάστασης, την z, οπότε η u παραμένει γραμμικός ελεγκτής.


Title: Re: [ΣΑΕ ΙΙ] Θέματα 2015
Post by: sotiristsar on June 07, 2016, 11:41:20 am
Για το Σεπτέμβριο του 2015 το 1 γ με την ευρωστία έχουμε καμία ιδέα ???


Title: Re: [ΣΑΕ ΙΙ] Θέματα 2015
Post by: nvog1993 on June 07, 2016, 12:09:36 pm
Για το Σεπτέμβριο του 2015 το 1 γ με την ευρωστία έχουμε καμία ιδέα ???
Στη σελ 61 έχω δώσει μία απάντηση που λογικά στέκει.


Title: Re: [ΣΑΕ ΙΙ] Θέματα 2015
Post by: Σα τανυστής on June 07, 2016, 13:41:52 pm
σεπτ15 θεμα 3 στο 1ο, στο χαρακτηριστικο πολυωνυμο και μετα εχει καποιος λιγο πιο αναλυτικη λυση ?

βασικα για να εξισωσω το χαρακτηριστικο του Α-ΒΚ με το s2+2ζωnn2 απλα λεω πρεπει να ισχυουν αυτες οι σχεσεις χωρις να διαλεξω κερδη? Ή το συνεχιζω?


Title: Re: [ΣΑΕ ΙΙ] Θέματα 2015
Post by: Dealan on June 07, 2016, 13:59:59 pm
Αφού ξέρεις ακριβώς τι ζ και ωn θες το συνεχίζεις.


Title: Re: [ΣΑΕ ΙΙ] Θέματα 2015
Post by: Μεταλλαγμένη Πάπια on June 07, 2016, 14:07:53 pm
Πως ακριβως το συνεχιζεις ομως; Τα γ,λ δε τα ξες και στο ελεγκτη θα χεις κ1,κ2 που επισης δε τα ξες. Θεωρητικα εχεις 4 αγνωστους και απο το ΧΠ βγαινουν 2 εξισωσεις. Εγω ετσι οπως το βλεπει πρεπει ειτε να το αφησουμε οπως ειναι και να πουμε πρεπει να ισχυουν οι 2 σχεσεις που βγαινουν η να θεσουμε αυθαιρετα τα κ1,κ2 κατι. και με βαση αυτα να βρουμε τα γ,λ (η και το αναποδο)


Title: Re: [ΣΑΕ ΙΙ] Θέματα 2015
Post by: Dealan on June 07, 2016, 14:16:11 pm
Τα γ, λ τα ξέρεις (90% σίγουρος πως ισχύει) οπότε βάζεις κ συναρτήσει αυτών.


Title: Re: [ΣΑΕ ΙΙ] Θέματα 2015
Post by: Μεταλλαγμένη Πάπια on June 07, 2016, 14:33:10 pm
Δε το λεει στην εκφωνηση οτι τα ξερουμε αλλα οκ, ετσι γινεται. Ειμαι σιγουρος οτι κανενας δε πηρε διευκρινιστικη απαντηση για το θεμα αυτο, καθως ειναι 3ο, και ο ροβι καθεται 2 ολοκληρα λεπτα για να απαντησει εωτησεις πριν παει σπιτι του.


Title: Re: [ΣΑΕ ΙΙ] Θέματα 2015
Post by: PanteGrv on June 07, 2016, 15:13:31 pm

γι αυτό να τα τσεκάρουμε όλα για απορίες πριν φύγει (αν και στην πρόοδο έκατσε πολυυ ώρα)


Title: Re: [ΣΑΕ ΙΙ] Θέματα 2015
Post by: Terzos on June 07, 2016, 19:43:28 pm
Ιούνιος 2015 Θέμα 4γ).
Συνεχίζουμε να ξέρουμε ότι a, b, c >=0?


Title: Re: [ΣΑΕ ΙΙ] Θέματα 2015
Post by: Giannis_X on June 07, 2016, 19:57:29 pm
ναι


Title: Re: [ΣΑΕ ΙΙ] Θέματα 2015
Post by: georgkonst on June 07, 2016, 20:23:11 pm
Όσον αφορά το θέμα 1γ του Σεπτεμβρίου 2015:
Κρατώντας τη γιαπουνοφ απο πριν καταλήγω στο Vdot = -x(x+v0) και θελω να δω αν θα ειναι < 0 ή όχι. Εδώ διίστανται οι απόψεις για τους εξής λόγους:
Σε προηγούμενη σελίδα κάποιος είπε μία λύση που το βγάζει πάντα εύρωστο ως: "Εγώ βρήκα το νέο ΣΙ αφού v(t) = v0 σταθερό, το οποίο είναι το χ* (0,v0/r0) . Με αλλαγή μεταβλητής, lyapunov και lasalle βγάζω το
z* (0,0) και άρα το x* (0,v0/r0) ασυμπτωτικά ευσταθές. Και αφού το v0 αρκούντως μικρό, με το v0/r0 ακόμα πιο μικρό, μπορώ να πω ότι το σύστημά μου είναι εύρωστο αφού θα ισορροπήσει σε ένα σημείο αρκετά κοντά στο αρχικό (0,0)."
Μία άλλη προσέγγιση είναι με ανισότητες:
θδο : x(x+v0)> 0 τότε |x|*|x+v0|>x(x+v0)
|x|^2 + |x|p0| > ..
> 0
αρα παντα ευρωστο

Μια άλλη όμως είναι με διερεύνηση προσήμου του χ1.
Αν χ1 > 0 τότε είναι εύρωστο για κάθε v0.
An x1 < 0 t;ote Vdot = -x1^2 - x1*v0 = -|x1|^2 + |x1|*V0 (από ορισμό απόλυτης τιμής ) < -|x1|^2 + |x1|*V0
Telika Vdot <= |x1|(r0 -|x1|) το οποίο είναι αρνητικό μόνο για |x1| > r0. Άρα είναι εύρωστο μόνο για |x1|>r0 (ή πρέπει να πω οτι αφού δεν είναι εύρωστο στο |x1|<r0 τότε δεν είναι γενικά εύρωστο).

Γνώμες και κανείς που να ξέρει τη σωστή απάντηση?


Title: Re: [ΣΑΕ ΙΙ] Θέματα 2015
Post by: Ancient on June 07, 2016, 20:37:00 pm
To 2o δε το καταλαβα.

Σχετικα με το 3ο, η εκφραση "δεν ειναι ευρωστο για |χ1|>v0" δεν είναι σωστή. Έπρεπε να πεις ότι είναι εύρωστο, επειδή αν |χ1|>v0 τότε V'<0 οπότε η λύση φθίνει συνεχώς μέχρι το |χ1|<v0, οποτε και τελικα το χ1 θα ειναι φραγμενο (αν παει να γινει |x1|>v0 τότε επειδή V'<0 θα γυρίσουμε πίσω). Ωστόσο αυτή η ανάλυση δε μας εξασφαλίζει ότι και η x2 θα είναι φραγμένη, οπότε μηδέν εις το πηλίκο.


Title: Re: [ΣΑΕ ΙΙ] Θέματα 2015
Post by: RFed the King on June 07, 2016, 20:38:11 pm
Ιουνιος 15
θεμα 1ο
Ε. σωστο δεν ειναι?
επειδη μια κατασταση ειναι  ενα σημειο (τιμη ) του χωρου καταστασεων?
ενω η ταξη ειναι το πληθος τον εξισωσεων καταστασης?


Title: Re: [ΣΑΕ ΙΙ] Θέματα 2015
Post by: status_quo on June 07, 2016, 20:48:46 pm
Ιουνιος 15
θεμα 1ο
Ε. σωστο δεν ειναι?
επειδη μια κατασταση ειναι  ενα σημειο (τιμη ) του χωρου καταστασεων?
ενω η ταξη ειναι το πληθος τον εξισωσεων καταστασης?

λαθος ειναι οσες εξισωσεις καταστασης εχεις τοσες μεταβλητες καταστασης εχει. Επισης, οχι οτι αποδεικνυει κατι για το συγκεκριμενο ερωτημα, απλα επειδη αναφερθηκαν οι εξισωσεις καταστασης για να ειναι ελαχιστης υλοποιησης δηλαδη να εκφραζεται το συστημα με τις λιγοτερες δυνατες μεταβλητες πρεπει να ειναι ελεγξιμο και παρατηρησιμο


Title: Re: [ΣΑΕ ΙΙ] Θέματα 2015
Post by: RFed the King on June 07, 2016, 20:53:07 pm
λαθος ειναι οσες εξισωσεις καταστασης εχεις τοσες μεταβλητες καταστασης εχει. Επισης, οχι οτι αποδεικνυει κατι για το συγκεκριμενο ερωτημα, απλα επειδη αναφερθηκαν οι εξισωσεις καταστασης για να ειναι ελαχιστης υλοποιησης δηλαδη να εκφραζεται το συστημα με τις λιγοτερες δυνατες μεταβλητες πρεπει να ειναι ελεγξιμο και παρατηρησιμο
????????


ναι αλλα εδω δεν λεει μεταβλητες καταστασης λεει καταστασεις σκετο


Title: Re: [ΣΑΕ ΙΙ] Θέματα 2015
Post by: lodi on June 07, 2016, 21:02:13 pm
To 2o δε το καταλαβα.

Σχετικα με το 3ο, η εκφραση "δεν ειναι ευρωστο για |χ1|>v0" δεν είναι σωστή. Έπρεπε να πεις ότι είναι εύρωστο, επειδή αν |χ1|>v0 τότε V'<0 οπότε η λύση φθίνει συνεχώς μέχρι το |χ1|<v0, οποτε και τελικα το χ1 θα ειναι φραγμενο (αν παει να γινει |x1|>v0 τότε επειδή V'<0 θα γυρίσουμε πίσω). Ωστόσο αυτή η ανάλυση δε μας εξασφαλίζει ότι και η x2 θα είναι φραγμένη, οπότε μηδέν εις το πηλίκο.

μόνο και μόνο που βγαίνει η παράγωγος της Lyapunov αρνητικά ημιορισμένη αυτό σημαίνει ότι έχω ευστάθεια , μπορεί όχι ασυμπτωτική όμως ευστάθεια . άρα και η χ2 είναι φραγμένη κάπου. Άρα είναι εύρωστο συνολικά το σύστημα .  όχι ;


Title: Re: [ΣΑΕ ΙΙ] Θέματα 2015
Post by: status_quo on June 07, 2016, 21:06:51 pm
????????


ναι αλλα εδω δεν λεει μεταβλητες καταστασης λεει καταστασεις σκετο
Το δευτερο δε το ειπα σε συνδεση με το θεμα που ρωτησες απλως επειδη πολλοι δε το προσεχουν και αν βαλει σ-λ μπορει να πεσει καμια τετοια μαλακιτσα το ανεφερα. για το 1.ε παντως με οσους το εχουμε δει ολοι λενε οτι ειναι λαθος, δε ξερω μπορει να εχεις και δικιο


Title: Re: [ΣΑΕ ΙΙ] Θέματα 2015
Post by: Giannis_X on June 07, 2016, 21:09:11 pm
μόνο και μόνο που βγαίνει η παράγωγος της Lyapunov αρνητικά ημιορισμένη αυτό σημαίνει ότι έχω ευστάθεια , μπορεί όχι ασυμπτωτική όμως ευστάθεια . άρα και η χ2 είναι φραγμένη κάπου. Άρα είναι εύρωστο συνολικά το σύστημα .  όχι ;
Η παράγωγος της V πρέπει να βγαίνει αρνητικά ορισμένη ή ημιορισμένη σε μια περιοχή-κύκλο που περιέχει το σημείο ισορροπίας. Εδώ βγαίνει αρνητική στο εξωτερικό του κύκλου |χ1|=v0 ο οποίος δεν περιέχει το ΣΙ, άρα δεν μπορείς να βγάλεις συμπέρασμα με το θεώρημα Lyapunov.


Title: Re: [ΣΑΕ ΙΙ] Θέματα 2015
Post by: Ancient on June 07, 2016, 21:14:18 pm
μόνο και μόνο που βγαίνει η παράγωγος της Lyapunov αρνητικά ημιορισμένη αυτό σημαίνει ότι έχω ευστάθεια , μπορεί όχι ασυμπτωτική όμως ευστάθεια . άρα και η χ2 είναι φραγμένη κάπου. Άρα είναι εύρωστο συνολικά το σύστημα .  όχι ;

Βγαίνει αρνητικά ορισμένη για |x1|>v0 όπότε τελικά θα έχουμε σίγουρα x1 στο (-v0, v0), δηλαδή φραγμένο κοντά στο 0. Όμως δεν έχουμε κάποιο στοιχείο που να συσχετίζει τη παράγωγο της lyapunov με το χ2 οπότε το αν είναι φραγμένο ή όχι είναι άγνωστο.


Title: Re: [ΣΑΕ ΙΙ] Θέματα 2015
Post by: lodi on June 07, 2016, 22:04:41 pm
Σ/Λ ιουνιους 2015 πρώτο:
σε μια υλοποίηση εξισώσεων κατάστασης ενός ΓΧΑ συστήματος οι ιδιοτιμές ταυτίζονται με τους πόλους της συνάρτησης μεταφοράς του .
Σωστό ή Λάθος και βασικότερο γιατί ;;


Title: Re: [ΣΑΕ ΙΙ] Θέματα 2015
Post by: Σα τανυστής on June 07, 2016, 22:07:37 pm
αυτο δεν ισχυει μονο για την ελαχιστη υλοποιηση? χωρις να ειμαι πολυ σιγουρος. Λ


Title: Re: [ΣΑΕ ΙΙ] Θέματα 2015
Post by: Giannis_X on June 07, 2016, 22:11:50 pm
Σ/Λ ιουνιους 2015 πρώτο:
σε μια υλοποίηση εξισώσεων κατάστασης ενός ΓΧΑ συστήματος οι ιδιοτιμές ταυτίζονται με τους πόλους της συνάρτησης μεταφοράς του .
Σωστό ή Λάθος και βασικότερο γιατί ;;
Λάθος είναι. Οι πόλοι της συνάρτησης μεταφοράς ταυτίζονται μόνο με τις ιδιοτιμές του παρατηρίσιμου κ ελέγξιμου μέρους ενός συστήματος , το οποίο προκύπτει από την αποδόμηση Kalman.


Title: Re: [ΣΑΕ ΙΙ] Θέματα 2015
Post by: lady_of_winter on June 07, 2016, 22:15:52 pm
Σ/Λ ιουνιους 2015 πρώτο:
σε μια υλοποίηση εξισώσεων κατάστασης ενός ΓΧΑ συστήματος οι ιδιοτιμές ταυτίζονται με τους πόλους της συνάρτησης μεταφοράς του .
Σωστό ή Λάθος και βασικότερο γιατί ;;

νομιζω ειναι λάθος, αυτό συμβαίνει μόνο όταν έχεις έλαχιστη υλοποιήση, δες στην τελευταία σελίδα του έκτου κεφαλαίου


Title: Re: [ΣΑΕ ΙΙ] Θέματα 2015
Post by: Escobar on September 05, 2016, 18:28:16 pm
Γενικά αξίζει κάποιος να ασχοληθεί με θέματα πριν το 2014? γιατί πριν το πάρει το μάθημα μόνος του ο ροβι, φαίνεται να ήταν διαφορετική η φιλοσοφία των θεμάτων.. so?


Title: Re: [ΣΑΕ ΙΙ] Θέματα 2015
Post by: vlad on September 05, 2016, 18:36:16 pm
Γενικά αξίζει κάποιος να ασχοληθεί με θέματα πριν το 2014? γιατί πριν το πάρει το μάθημα μόνος του ο ροβι, φαίνεται να ήταν διαφορετική η φιλοσοφία των θεμάτων.. so?
Να ασχοληθεις μονο με του Ροβι.


Title: Re: [ΣΑΕ ΙΙ] Θέματα 2015
Post by: xameno kormi on September 07, 2016, 18:10:52 pm
φεβρ.'15, θεμα 4 ερωτημα Α  είδα την λύση που έχει ανέβει στα downloads και ήθελα να ρωτησω , δεν μπορούμε αντι να το κανουμε ετσι να πουμε οτι τα x1(t)=2sin(t) και x2(t)=2cos(t) αποτελουν λυση του συστηματος και άρα θα είμαστε παντα πανω στον οριακο κυκλο (x1)^2 + (x2)^2 = 4 ? 


Title: Re: [ΣΑΕ ΙΙ] Θέματα 2015
Post by: matzaris on September 09, 2016, 09:56:27 am
Στο θέμα 4γ του Ιουνίου του 2015, παρόλο που είδα την λύση που ανέβηκε σε προηγούμενο post δεν την κατάλαβα. Μήπως κάποιος θα μπορούσε να την  εξηγήσει περισσότερο; Για τα ρ1 και ρ2 εννοεί ότι είναι οι ρίζες του Χ.Π; Help!!!


Title: Re: [ΣΑΕ ΙΙ] Θέματα 2015
Post by: Μεταλλαγμένη Πάπια on September 09, 2016, 10:43:53 am
φεβρ.'15, θεμα 4 ερωτημα Α  είδα την λύση που έχει ανέβει στα downloads και ήθελα να ρωτησω , δεν μπορούμε αντι να το κανουμε ετσι να πουμε οτι τα x1(t)=2sin(t) και x2(t)=2cos(t) αποτελουν λυση του συστηματος και άρα θα είμαστε παντα πανω στον οριακο κυκλο (x1)^2 + (x2)^2 = 4 ?  

Πρέπει να αποδείξεις ότι αυτή είναι μοναδική λύση πράγμα που θεωρώ θέλει πολλή περισσότερη δουλειά από αυτό που έκανα εγώ. Θεωρητικά είναι σωστό όμως.


Title: Re: [ΣΑΕ ΙΙ] Θέματα 2015
Post by: Schro on September 12, 2016, 18:08:43 pm
μετά τον πανικό απαντήσεων που ακολούθησε το τελευταίο post, έχει λύσει κάποιος το 3 ερώμα στο θέμα 4 του ιουνίου 2015 να διασταυρώσουμε αποτελέσματα;


Title: Re: [ΣΑΕ ΙΙ] Θέματα 2015
Post by: Escobar on September 12, 2016, 19:27:58 pm
μετά τον πανικό απαντήσεων που ακολούθησε το τελευταίο post, έχει λύσει κάποιος το 3 ερώμα στο θέμα 4 του ιουνίου 2015 να διασταυρώσουμε αποτελέσματα;


τι διαδικασία ακολούθησες?


Title: Re: [ΣΑΕ ΙΙ] Θέματα 2015
Post by: Schro on September 12, 2016, 19:37:46 pm
τι διαδικασία ακολούθησες?

προχώρησα με δυναμική ανάδραση.
θεώρησα ότι z1 = y(έξοδος του συστήματος) και πρόσθεσα ένα ολοκληρωτή m  = z1 - δ.
θεώρησα τον ελεγκτή u = -k1z1 - k2z2 -ki*m και έκανα αντικατάσταση στην z = Ax + Bu.
μετά από πράξεις κατέληξα σε έναν πίνακα 3x3 και είπα ότι θέλω ο πίνακας αυτός να οδηγεί σε ευσταθές σύστημα. βρήκα το χαρακτηριστικό πολ/μο και προσπάθησα με Routh και με τις ανισώσεις που δίνει να βρω κάποιες σχέσεις που να περιέχουν μόνο γνωστές παραμέτρους p1,p2 για τα κέρδη του ελεγκτή. αυτό το τελευταίο δεν το κατάφερα. σαν διαδικασία είναι σωστό;


Title: Re: [ΣΑΕ ΙΙ] Θέματα 2015
Post by: Escobar on September 12, 2016, 20:02:49 pm
προχώρησα με δυναμική ανάδραση.
θεώρησα ότι z1 = y(έξοδος του συστήματος) και πρόσθεσα ένα ολοκληρωτή m  = z1 - δ.
θεώρησα τον ελεγκτή u = -k1z1 - k2z2 -ki*m και έκανα αντικατάσταση στην z = Ax + Bu.
μετά από πράξεις κατέληξα σε έναν πίνακα 3x3 και είπα ότι θέλω ο πίνακας αυτός να οδηγεί σε ευσταθές σύστημα. βρήκα το χαρακτηριστικό πολ/μο και προσπάθησα με Routh και με τις ανισώσεις που δίνει να βρω κάποιες σχέσεις που να περιέχουν μόνο γνωστές παραμέτρους p1,p2 για τα κέρδη του ελεγκτή. αυτό το τελευταίο δεν το κατάφερα. σαν διαδικασία είναι σωστό;

νομίζω πως ναι, εγώ στο μυαλό μου είχα γραμμική ανάδραση στην αρχή αλλά που είδα πιο πίσω λένε ότι δεν γίνεται. Ποιος είναι ο 3x3 που καταλήγεις?


Title: Re: [ΣΑΕ ΙΙ] Θέματα 2015
Post by: Schro on September 12, 2016, 20:12:23 pm
νομίζω πως ναι, εγώ στο μυαλό μου είχα γραμμική ανάδραση στην αρχή αλλά που είδα πιο πίσω λένε ότι δεν γίνεται. Ποιος είναι ο 3x3 που καταλήγεις?

|s                                        -1                                             0|
|acos(x*1)+ck1              s+b+ck2                                  ck2|
|-1                                        0                                             s|


μετά το Routh γίνεται το μπάχαλο!


Title: Re: [ΣΑΕ ΙΙ] Θέματα 2015
Post by: Escobar on September 12, 2016, 20:16:06 pm
έχεις κάποια πρόσημα λάθος, αλλά γενικά αυτή είναι η μορφή. Δες το πιο κάτω ποστ, και εγώ στα ίδια έφτασα (και μάλλον αυτή είναι η λύση)



Εγώ έτσι θα το έκανα.


Title: Re: [ΣΑΕ ΙΙ] Θέματα 2015
Post by: Schro on September 13, 2016, 17:02:46 pm
Σεπτ.2015, ξέρουμε ποια είναι τα σωστά Σ-Λ?

έχετε λύσεις για αυτό μήπως;
έσκαψα στις προηγούμενες σελίδες αλλά δεν τα βρήκα κάπου.
τα β,γ,δ τα βγάζω σωστά και το α και ε λάθος εγώ αλλά παίζει να τα έχω σκοτώσει.

edit απάντηση:

ολα Σ, ειδα το γραπτο σμρ



Title: Re: [ΣΑΕ ΙΙ] Θέματα 2015
Post by: Γρηγόρης on September 13, 2016, 19:43:33 pm
Έχει λύσει κανένας 4ο θεμα ιουνιος 2015; Έχω σκαλώσει απο΄την αρχή άσχημα, ορίζω μεταβλητές κατάστασης x1= theta kai x2= thetadot αλλά και πάλι δεν λέει να μου βγει η γραμμικοποίηση  :'( :'( :'( :'(


Title: Re: [ΣΑΕ ΙΙ] Θέματα 2015
Post by: Schro on September 13, 2016, 21:20:18 pm
Έχει λύσει κανένας 4ο θεμα ιουνιος 2015; Έχω σκαλώσει απο΄την αρχή άσχημα, ορίζω μεταβλητές κατάστασης x1= theta kai x2= thetadot αλλά και πάλι δεν λέει να μου βγει η γραμμικοποίηση  :'( :'( :'( :'(

λοιπόν αφού ορίσεις αυτές τις μεταβλητές όπως λες και εσύ μετά βρίσκεις το σημείο ισορροπίας το οποίο είναι
(x1*,x2*,u*) = (x1*,0,a/c sinx1*).
Μετά κάνεις αλλαγή μεταβλητών ώστε να κάνεις το 0 σημείο ισορροπίας και προχωράς σε γραμμικοποίηση. δεν είναι κάτι extreme έχει λύσει και τέτοια παραδείγματα μέσα στην τάξη αν θες να δεις από τις φετινές σημειώσεις.
το γ ερώτημα το συζητήσαμε εδώ και χθες. κάνεις δυναμική ανάδραση, σου βγαίνει ένας πίνακας 3x3 και βρίσκεις το χαρακτηριστικό πολυώνυμο και με Routh μπορείς να βγάλεις ανισώσεις για τα στοιχεία του ελεγκτή σου.


Title: Re: [ΣΑΕ ΙΙ] Θέματα 2015
Post by: Schro on September 13, 2016, 21:55:58 pm
σεπτέμβριος 2015 στο θέμα 3
στο (α) έχω k1 = 4/λ , k2 = (2*sqrt(2) *λ - 4γ)/2λ2
στο ερώτημα (γ) πρέπει να πούμε ότι για τιμές του l κοντά στο μηδέν θα έχω μεγάλο k1 που θα προκαλεί μεγάλες διακυμάνσεις και το σύστημα λογικά δεν θα μπορεί να το ακολουθήσει, σωστά;

στο ερώτημα β πως απαντάμε;

επίσης στα ίδια θέματα στο θέμα 1 για την ευστάθεια του σ.ι. που ζητάει στο 2ο ερώτημα μπορούμε να προχωρήσουμε με όποιον τρόπο θέλουμε; (δηλαδή μπορώ να κάνω και γραμμικοποίηση γύρω από το σ.ι.?)

Sorry για το σπαμάρισμα αλλά κάπου θέλω να τα συζητήσω και εγώ!  ::)


Title: Re: [ΣΑΕ ΙΙ] Θέματα 2015
Post by: Schro on September 14, 2016, 11:27:19 am
άλλη ερώτηση: αν βρω ότι ένα  μη-γραμμικό σύστημα έχει ένα μόνο σημείο ισορροπίας, και μετά με γραμμικοποίηση γύρω από το σημείο ισορροπίας βρώ ότι το γραμ/νο σύστημα είναι ασυμπτωτικά ευσταθές. μετά μπορώ για το αρχικό μη-γραμμικό σύστημα να πω ότι -εφόσον έχω μόνο 1 σ.ι. - είναι ολικά ασυμπτωτικά ευσταθές?

βασικά σκεφτόμουν να προχωρήσω έτσι την μελέτη ευστάθειας στο 1ο θέμα σεπτ. 2015.
το είδε κανένας;


Title: Re: [ΣΑΕ ΙΙ] Θέματα 2015
Post by: jimPster on September 14, 2016, 11:30:58 am
λαθος διαπιστωση γτ οταν κανεις γραμμικοποιηση λες οτι κινεισαι κοντα στην περιοχη του ΣΙ και εσυ καταληγεις οτι απο οπουδηποτε και ν ξεκινησω (ολικα) θα συγκλινω κτλ

ειναι τοπικο το λεει στις σημειωσεις Ροβ

Παντως με γραμμικοποιηση δεν μπορεις να αποδειξεις ποτε ολικη  ευσταθεια γι αυτο το λογο που σ πα, μονο με καταλληλη lyapunov και αν ισχυει βεβαια


Title: Re: [ΣΑΕ ΙΙ] Θέματα 2015
Post by: orestisk4 on September 14, 2016, 11:34:41 am
Βασικά διάβασα κάπου ότι η γραμμικοποίηση ισχύει μόνο τοπικά. Συνεπώς, αν βρεις ότι το σύστημα είναι π.χ. ασυμπτωτικά ευσταθές τότε θα είναι τοπικά.


Title: Re: [ΣΑΕ ΙΙ] Θέματα 2015
Post by: Schro on September 14, 2016, 11:40:34 am
thank you both! και εγώ επειδή είχα δει ότι με γραμμικοποίηση περιορίζεσαι στην γειτονιά του σ.ι. ρώτησα, απλά σκεφτόμουν μήπως δεδομένου ότι δεν έχεις κάποιο άλλο σ.ι. μπορείς να το γενικεύσεις.
στο 1ο θέμα σεπτ.2015 πως πήγατε εσείς; με Lyapunov?


Title: Re: [ΣΑΕ ΙΙ] Θέματα 2015
Post by: xameno kormi on September 14, 2016, 11:58:41 am
αν δοκιμασεις την V = 1/2 * γ * x12 + 1/2 x22 καταληγεις στην V'=  -γ*x12 σωστα ?


Title: Re: [ΣΑΕ ΙΙ] Θέματα 2015
Post by: Schro on September 14, 2016, 12:00:58 pm
αν δοκιμασεις την V = 1/2 * γ * x12 + 1/2 x22 καταληγεις στην V'=  -γ*x12 σωστα ?

δεν την δοκίμασα ακόμα αυτήν. εγώ το έκανα με γραμμικοποίηση γύρω από το σ.ι. γιατί με την απλή τετραγωνική Lyapunov δεν μου βγήκε. θα το δω σε λίγο με αυτήν!  :)


Title: Re: [ΣΑΕ ΙΙ] Θέματα 2015
Post by: xameno kormi on September 14, 2016, 12:03:59 pm
okk νομιζω βγαινει και ετσι παντως και δινει οτι και το γ > 0  οποτε δεν εχεις θεμα :)


Title: Re: [ΣΑΕ ΙΙ] Θέματα 2015
Post by: xameno kormi on September 14, 2016, 12:26:48 pm
για το πρωτο θεμα παλι σεπτ'15 το (γ) ερωτημα το εκανε κανεις ??


Title: Re: [ΣΑΕ ΙΙ] Θέματα 2015
Post by: Λήσταρχος Γιαγκούλας on September 14, 2016, 12:46:04 pm
αν δοκιμασεις την V = 1/2 * γ * x12 + 1/2 x22 καταληγεις στην V'=  -γ*x12 σωστα ?

Συμφωνώ φίλε, το ίδιο έκανα και γω.


Title: Re: [ΣΑΕ ΙΙ] Θέματα 2015
Post by: Schro on September 14, 2016, 12:52:04 pm
για το πρωτο θεμα παλι σεπτ'15 το (γ) ερωτημα το εκανε κανεις ??

βρήκα αυτό (https://www.thmmy.gr/smf/index.php?topic=32998.msg1130473#msg1130473) στα προηγούμενα post. δεν ξέρω αν είναι σωστό είχα σκοπό να το δω αργότερα.


Title: Re: [ΣΑΕ ΙΙ] Θέματα 2015
Post by: xameno kormi on September 14, 2016, 13:44:34 pm
οποτε μαλλον χρειαζεται να αποδειξεις οτι τα χ1,χ2 φραγμενα φανταζομαι..θα το ξανακοιταξω κ αργοτερα κ γω μπας και βγει καμια ακρη γτ με μπερδεψε !


Title: Re: [ΣΑΕ ΙΙ] Θέματα 2015
Post by: Schro on September 14, 2016, 13:50:05 pm
οποτε μαλλον χρειαζεται να αποδειξεις οτι τα χ1,χ2 φραγμενα φανταζομαι..θα το ξανακοιταξω κ αργοτερα κ γω μπας και βγει καμια ακρη γτ με μπερδεψε !

άμα βγάλεις άκρη πες το μας και εμάς!  :)


Title: Re: [ΣΑΕ ΙΙ] Θέματα 2015
Post by: Schro on September 15, 2016, 11:22:41 am
άλλη ερώτηση, στο θέμα 2 ιουνίου 2015 μου βγαίνει με την γραμμικοποίηση διαγώνιος πίνακας για το γραμμικοποιημένο σύστημα
Α = [-1 0; 0 -1] άρα οι ιδιοτιμές είναι τα στοιχεία της διαγωνίου, άρα το γραμμικοποιημένο σύστημα δεν θα είναι εκθετικά ευσταθές; μπορώ μετά για το μη-γραμμικοποιημένο να πω ότι είναι τοπικά ασυμπτωτικά ευσταθές ή έχω κάνει (πάλι) λάθος στις πράξεις;


Title: Re: [ΣΑΕ ΙΙ] Θέματα 2015
Post by: jimPster on September 15, 2016, 11:26:04 am
σωστα τα λες εκθετικη ευσταθεια ειναι ασυμπτωτικη ευσταθεια


Title: Re: [ΣΑΕ ΙΙ] Θέματα 2015
Post by: Schro on September 15, 2016, 11:27:51 am
σωστα τα λες εκθετικη ευσταθεια ειναι ασυμπτωτικη ευσταθεια

οκ, οπότε απαντάω ότι το γραμμικοποιημένο είναι εκθετικά ευσταθές, άρα το μη-γραμ/νο θα είναι τοπικά (τουλάχιστον) ασυμπτωτικά ευσταθές και είμαι οκ!
thanks!


Title: Re: [ΣΑΕ ΙΙ] Θέματα 2015
Post by: mitsoschelsea on September 15, 2016, 11:48:42 am
οκ, οπότε απαντάω ότι το γραμμικοποιημένο είναι εκθετικά ευσταθές, άρα το μη-γραμ/νο θα είναι τοπικά (τουλάχιστον) ασυμπτωτικά ευσταθές και είμαι οκ!
thanks!

Ποιό είναι το μη-γραμμικοποιημένο και γιατί να το αναφέρουμε;;


Title: Re: [ΣΑΕ ΙΙ] Θέματα 2015
Post by: xameno kormi on September 15, 2016, 12:21:18 pm
να ρωτησω κατι γενικο, θεμα 4 ιουνιος του '15
οταν παιρνει την μερικη παραγωγο και βγαζει -αcos(x1) δεν το παραγωγιζουμε σαν συνθετη συναρτηση επειδη το x1 ανηκει στο R αφου και θ ανηκει στο R ?


Title: Re: [ΣΑΕ ΙΙ] Θέματα 2015
Post by: Schro on September 15, 2016, 12:37:04 pm
να ρωτησω κατι γενικο, θεμα 4 ιουνιος του '15
οταν παιρνει την μερικη παραγωγο και βγαζει -αcos(x1) δεν το παραγωγιζουμε σαν συνθετη συναρτηση επειδη το x1 ανηκει στο R αφου και θ ανηκει στο R ?

αν κατάλαβα τι ρωτάς, είχα την ίδια απορία πριν λίγο. γενικά παίρνεις την μερική παράγωγο ως προς z1 και όχι ως προς t.


Title: Re: [ΣΑΕ ΙΙ] Θέματα 2015
Post by: xameno kormi on September 15, 2016, 12:51:22 pm
αν κατάλαβα τι ρωτάς, είχα την ίδια απορία πριν λίγο. γενικά παίρνεις την μερική παράγωγο ως προς z1 και όχι ως προς t.

αα ναι σωστα εχεις δικιο , τνξ !!


Title: Re: [ΣΑΕ ΙΙ] Θέματα 2015
Post by: Schro on September 15, 2016, 12:52:51 pm
γενικά στου ιουνίου του 2015 που πρέπει να πάμε με δυναμική ανάδραση. αφού σχηματίσουμε τον Α που είναι 3x3 μετά για να σταθεροποιηθεί η έξοδος ασυμπτωτικά στο δ πρέπει το σύστημα να είναι ευσταθές; αρκεί δηλαδή αυτό;


Title: Re: [ΣΑΕ ΙΙ] Θέματα 2015
Post by: Escobar on September 15, 2016, 13:05:23 pm
γενικά στου ιουνίου του 2015 που πρέπει να πάμε με δυναμική ανάδραση. αφού σχηματίσουμε τον Α που είναι 3x3 μετά για να σταθεροποιηθεί η έξοδος ασυμπτωτικά στο δ πρέπει το σύστημα να είναι ευσταθές; αρκεί δηλαδή αυτό;

κάτι τέτοιο λογικά, δε μπορείς να κάνεις και κάτι άλλο. Απλά έχει αρκετή δουλειά μετά στο Routh


Title: Re: [ΣΑΕ ΙΙ] Θέματα 2015
Post by: Schro on September 15, 2016, 13:11:16 pm
κάτι τέτοιο λογικά, δε μπορείς να κάνεις και κάτι άλλο. Απλά έχει αρκετή δουλειά μετά στο Routh

ναι θυμάμαι το είχα δει προχθες νομίζω. απλά το κοιτούσα και σκεφτόμουν αν αρκεί η ευστάθεια.
σε άνω τριγωνικό πίνακα το χαρακτηριστικό πολ/μο είναι το γινόμενο των στοιχεών τις διαγωνίου;


Title: Re: [ΣΑΕ ΙΙ] Θέματα 2015
Post by: Escobar on September 15, 2016, 13:43:11 pm
σε άνω τριγωνικό πίνακα το χαρακτηριστικό πολ/μο είναι το γινόμενο των στοιχεών τις διαγωνίου;

ναι


Title: Re: [ΣΑΕ ΙΙ] Θέματα 2015
Post by: Schro on September 15, 2016, 14:09:18 pm
στον σεπτέβριο του 2015 στο θέμα 3 με ποια λογική επιλέγουμε τους ενεργοποιητές είπαμε; το λ προφανώς θα πρέπει να είναι διάφορο του μεδεν επειδή το χρησιμοποιώ σε παρανομαστή των ελεγκτών. για το γ έχω κάποιον περιορισμό;

επίσης στην ίδια χρονιά για το θέμα 1γ με την ευρωστία (  :o  ) έχουμε καμιά αποδεκτή και κατανοητή λύση;


Title: Re: [ΣΑΕ ΙΙ] Θέματα 2015
Post by: nash on September 15, 2016, 16:28:58 pm
παιδια επειδη ειμαι αρκετα  :o  :o  :o  :o  μπορεί καποιος/καποια με λιγα λογια να μ πει ,ποτε καποιος επιλέγει το Lyapunov και ποτε πηγαινει με Lasalle?με ποιο κριτηριο? διαβασα τους ορισμους και δεν εβγαλα ακρη  :(


Title: Re: [ΣΑΕ ΙΙ] Θέματα 2015
Post by: xameno kormi on September 15, 2016, 16:43:10 pm
παιδια επειδη ειμαι αρκετα  :o  :o  :o  :o  μπορεί καποιος/καποια με λιγα λογια να μ πει ,ποτε καποιος επιλέγει το Lyapunov και ποτε πηγαινει με Lasalle?με ποιο κριτηριο? διαβασα τους ορισμους και δεν εβγαλα ακρη  :(
το Lassale ειναι για οταν σου βγει η Lyapunov αρνητικα ημιορισμενη (δηλαδη V '<=0 αλλα θες να δεις αν ειναι ασυμπτωτικη η ευσταθεια κι αυτο το βλεπεις μεσα απο το Lassale και τις εξισωσεις καταστασης (αν βγαινει οτι το 0 ειναι το μεγαλυτερο αμεταβλητο στο οποιο συγκλινεις)
δες στις σημειωσεις τις φετινες εκει που λεει για το Lasalle εχει απο κατω ενα παραδειγμα


στο θεμα 2 ερωτημα β του ιουνιου '15 στον πινακα Α μηπως εχει γινει λαθος η διαφοριση ; δεν παει [ δx1'/δx1  δx1'/δx2  ; δx2'/δx1 δx2'/δx2 ] o πινακας στην ουσια; παλι το ιδιο βγαινει οταν βαλεις τα Σ.Ι αλλα ρωταω γενικα μηπως το καταλαβα λαθος εγω στην διαφοριση


Title: Re: [ΣΑΕ ΙΙ] Θέματα 2015
Post by: matzaris on September 15, 2016, 17:34:31 pm
Για τον Σεπτέμβριο του 2015,θέμα 1γ, κάποιος να προτείνει κάτι για το αν το σύστημα είναι εύρωστο...  :-\


Title: Re: [ΣΑΕ ΙΙ] Θέματα 2015
Post by: Μπιγκόνια on September 15, 2016, 17:53:40 pm
Σεπτέμβριος 15 τα ΣΛ πως είναι?


Title: Re: [ΣΑΕ ΙΙ] Θέματα 2015
Post by: nash on September 15, 2016, 18:05:46 pm

στο θεμα 2 ερωτημα β του ιουνιου '15 στον πινακα Α μηπως εχει γινει λαθος η διαφοριση ; δεν παει [ δx1'/δx1  δx1'/δx2  ; δx2'/δx1 δx2'/δx2 ] o πινακας στην ουσια; παλι το ιδιο βγαινει οταν βαλεις τα Σ.Ι αλλα ρωταω γενικα μηπως το καταλαβα λαθος εγω στην διαφοριση
Σωστα  το καταλαβες :) ευχαριστω για την απαντηση  ;)


Title: Re: [ΣΑΕ ΙΙ] Θέματα 2015
Post by: marijuanna on September 15, 2016, 19:05:42 pm
Για τον Σεπτέμβριο του 2015,θέμα 1γ, κάποιος να προτείνει κάτι για το αν το σύστημα είναι εύρωστο...  :-\

Δεν ειμαι σιγουρη αλλα το συζητησαμε με καποια παιδια και ισως ειναι καπως ετσι. (επισυναπτω φωτο)


Title: Re: [ΣΑΕ ΙΙ] Θέματα 2015
Post by: Schro on September 15, 2016, 22:44:36 pm
Δεν ειμαι σιγουρη αλλα το συζητησαμε με καποια παιδια και ισως ειναι καπως ετσι. (επισυναπτω φωτο)

εφόσον στο χ_2 δεν έχεις κάποια πρόσθετη διαταραχή και το χ_1 στο οποίο έχεις πρόσθετη διαταραχή είναι ομοιόμορφα τελικώς φραγμένο δεν πρέπει να πούμε ότι το σύστημα είναι εύρωστο;


Title: Re: [ΣΑΕ ΙΙ] Θέματα 2015
Post by: Λήσταρχος Γιαγκούλας on September 15, 2016, 23:27:12 pm
Δεν ειμαι σιγουρη αλλα το συζητησαμε με καποια παιδια και ισως ειναι καπως ετσι. (επισυναπτω φωτο)

Μπορεί κάποιος να μου εξηγήσει στη λύση της Μarijuanna
για ποιό λόγο μπορείς να πείς σίγουρα πως

dV/dt<=0 ?


Title: Re: [ΣΑΕ ΙΙ] Θέματα 2015
Post by: Kodi on January 09, 2017, 22:47:12 pm
για το θέμα 4 στο ερώτημα γ, (ιουνιος 2015) διάβασα ότι θέλει δυναμική ανάδραση καταστάσεων, όμως δεν καταλαβαίνω πως θα χρησιμοποιηθούν οι συνθήκες για τα p1 και p2 που μας δίνει. έχει κανείς καμιά ιδέα;


Title: Re: [ΣΑΕ ΙΙ] Θέματα 2015
Post by: AckermanMik on January 09, 2017, 22:48:45 pm
για το θέμα 4 στο ερώτημα γ, διάβασα ότι θέλει συναμική ανάδραση καταστάσεων, όμως δεν καταλαβαίνω πως θα χρησιμοποιηθούν οι συνθήκες για τα p1 και p2 που μας δίνει. έχει κανείς καμιά ιδέα;

Ρουθ για ευαστάθεια!


Title: Re: [ΣΑΕ ΙΙ] Θέματα 2015
Post by: adiaforos on February 10, 2017, 19:06:24 pm
Σεπτέμβριος 2015 θέμα πρώτο ερώτημα για..ρωτάει αν το σύστημα είναι ευρωστο..το είδα κάπου στις διαφάνειες και στις σημειώσεις αλλά δεν κατάλαβα καθόλου πως θα έπρεπε να αντιμετωπιστει το θέμα...υπάρχει καμμια φιλική συμβουλη?


Title: Re: [ΣΑΕ ΙΙ] Θέματα 2015
Post by: Escobar on February 10, 2017, 19:41:30 pm
Σεπτέμβριος 2015 θέμα πρώτο ερώτημα για..ρωτάει αν το σύστημα είναι ευρωστο..το είδα κάπου στις διαφάνειες και στις σημειώσεις αλλά δεν κατάλαβα καθόλου πως θα έπρεπε να αντιμετωπιστει το θέμα...υπάρχει καμμια φιλική συμβουλη?


Δες την απάντηση του nvog πιο πίσω:

Εγώ βρήκα το νέο ΣΙ αφού v(t) = v0 σταθερό, το οποίο είναι το χ* (0,v0/r0) . Με αλλαγή μεταβλητής, lyapunov και lasalle βγάζω το
z* (0,0) και άρα το x* (0,v0/r0) ασυμπτωτικά ευσταθές. Και αφού το v0 αρκούντως μικρό, με το v0/r0 ακόμα πιο μικρό, μπορώ να πω ότι το σύστημά μου είναι εύρωστο αφού θα ισορροπήσει σε ένα σημείο αρκετά κοντά στο αρχικό (0,0).


Title: Re: [ΣΑΕ ΙΙ] Θέματα 2015
Post by: s0r0n on February 12, 2017, 18:24:11 pm
Ιουνιος 2015,στο τριτο θεμα,με βαση το Lassalle νομιζω καταληγουμε σε τοπικα ασυμπτωτικη ευσταθεια ενω ζηταει ολικη
Επισης εχει κανεις τη συνεχεια απο το τεταρτο θεμα?


Title: Re: [ΣΑΕ ΙΙ] Θέματα 2015
Post by: matzaris on February 12, 2017, 19:23:48 pm
Ιουνιος 2015,στο τριτο θεμα,με βαση το Lassalle νομιζω καταληγουμε σε τοπικα ασυμπτωτικη ευσταθεια ενω ζηταει ολικη
Ίσως επειδή είναι το μοναδικό σημείο ισορροπίας


Title: Re: [ΣΑΕ ΙΙ] Θέματα 2015
Post by: arisalex on February 13, 2017, 01:58:08 am
Ίσως επειδή είναι το μοναδικό σημείο ισορροπίας
Νομίζω από την στιγμή που δεν παίρνουμε κάποιον περιορισμό(κοντά στο 0,0) μιλάμε για ολική ευστάθεια.


Title: Re: [ΣΑΕ ΙΙ] Θέματα 2015
Post by: geralt on February 13, 2017, 15:39:05 pm
Όταν ζητά ελεγκτή ανάδρασης εξόδου που να παει τις ιδιοτιμες κάπου. Μπορείς να πάρεις u=-kx σκέτο  επειδή δεν σου ζητά να μεταφέρεις την έξοδο σε κάποια άλλη τιμή; Ή πάντα πρέπει να βρεις και το Kr ?

Επίσης Ιούνιο 2015 θέμα 3ο γ ερώτημα, υποθέτω παίρνουμε ελεγκτή με u=-kx+Krδ ? αφού όταν λεει να πάει στο δ εννοεί r(t)=δ και Kr=-1/(C(A-BK)^-1B)?


Title: Re: [ΣΑΕ ΙΙ] Θέματα 2015
Post by: geralt on February 13, 2017, 20:21:32 pm
Σεπτ 2015 θεμα 1γ πως λεμε αν ειναι ευρωστο ή οχι;

edit:
Εγώ βρήκα το νέο ΣΙ αφού v(t) = v0 σταθερό, το οποίο είναι το χ* (0,v0/r0) . Με αλλαγή μεταβλητής, lyapunov και lasalle βγάζω το
z* (0,0) και άρα το x* (0,v0/r0) ασυμπτωτικά ευσταθές. Και αφού το v0 αρκούντως μικρό, με το v0/r0 ακόμα πιο μικρό, μπορώ να πω ότι το σύστημά μου είναι εύρωστο αφού θα ισορροπήσει σε ένα σημείο αρκετά κοντά στο αρχικό (0,0).


Title: Re: [ΣΑΕ ΙΙ] Θέματα 2015
Post by: geralt on February 13, 2017, 21:42:28 pm
Παιδιά είναι ιδέα μου ή η λύση αυτή από τα λυμένα (σεπτ 2015) που έχουν ανέβει σε προηγούμενα ποστ είναι τελείως λάθος; Δεν έχει ξεχάσει να πολλαπλασιάσει με το B τον ελεγκτή; Όταν k=-kx => xdot=(A-BK)x


Title: Re: [ΣΑΕ ΙΙ] Θέματα 2015
Post by: iason1907 on February 13, 2017, 21:54:37 pm
Παιδιά είναι ιδέα μου ή η λύση αυτή από τα λυμένα που έχουν ανέβει σε προηγούμενα ποστ είναι τελείως λάθος; Δεν έχει ξεχάσει να πολλαπλασιάσει με το B τον ελεγκτή; Όταν k=-kx => xdot=(A-BK)x
εγώ βγάζω λύσεις με τα γ,λ μέσα που σε αυτό που έβαλες δεν υπάρχουν.


Title: Re: [ΣΑΕ ΙΙ] Θέματα 2015
Post by: heavy melon on February 13, 2017, 22:03:46 pm
Παιδιά είναι ιδέα μου ή η λύση αυτή από τα λυμένα που έχουν ανέβει σε προηγούμενα ποστ είναι τελείως λάθος; Δεν έχει ξεχάσει να πολλαπλασιάσει με το B τον ελεγκτή; Όταν k=-kx => xdot=(A-BK)x

ναι λάθος είναι


Title: Re: [ΣΑΕ ΙΙ] Θέματα 2015
Post by: geralt on February 13, 2017, 22:19:13 pm
εγώ βγάζω λύσεις με τα γ,λ μέσα που σε αυτό που έβαλες δεν υπάρχουν.

Δεν ήταν δική μου λύση


Title: Re: [ΣΑΕ ΙΙ] Θέματα 2015
Post by: iason1907 on February 13, 2017, 22:20:48 pm
Δεν ήταν δική μου λύση
ενταξει μην αγχώνεσαι.

Η λύση που έβαλες είναι για συγκεκριμένα γλ και αν θυμάμαι καλά αυτά που θέλει στο β


Title: Re: [ΣΑΕ ΙΙ] Θέματα 2015
Post by: mitsoschelsea on February 14, 2017, 14:46:13 pm
Έχουμε λύσεις Σεπτεμβρίου 2015;


Title: Re: [ΣΑΕ ΙΙ] Θέματα 2015
Post by: iason1907 on February 14, 2017, 16:12:39 pm
Έχουμε λύσεις Σεπτεμβρίου 2015;
  υπάρχουν ανεβασμένες λίγο πιο πίσω.
Το Θέμα 3 (https://s12.postimg.org/z1b0ktq99/IMG_0369.jpg) (αν και το έχω κάνει αρκετά πρόχειρα)


Title: Re: [ΣΑΕ ΙΙ] Θέματα 2015
Post by: DarkPassenger on February 14, 2017, 16:13:56 pm
  υπάρχουν ανεβασμένες λίγο πιο πίσω.
Το Θέμα 3 (https://s12.postimg.org/z1b0ktq99/IMG_0369.jpg) (αν και το έχω κάνει αρκετά πρόχειρα)
υπάρχουν και τα σωστό/λαθος απο το θεμα 2?


Title: Re: [ΣΑΕ ΙΙ] Θέματα 2015
Post by: geralt on February 14, 2017, 16:18:48 pm
  υπάρχουν ανεβασμένες λίγο πιο πίσω.
Το Θέμα 3 (https://s12.postimg.org/z1b0ktq99/IMG_0369.jpg) (αν και το έχω κάνει αρκετά πρόχειρα)

ωραίος, έτσι το έλυσα. Τα ίδια βρήκαμε.


Title: Re: [ΣΑΕ ΙΙ] Θέματα 2015
Post by: olgatsim on February 14, 2017, 16:22:09 pm
υπάρχουν και τα σωστό/λαθος απο το θεμα 2?

Όλα σωστά...


Title: Re: [ΣΑΕ ΙΙ] Θέματα 2015
Post by: DarkPassenger on February 14, 2017, 16:26:39 pm
Όλα σωστά...
το (γ) που λεει στον ελεγχο ΓΧΑ συστηματων με αναδραση εξοδου και χρηση παρατηρητη, η σχεδίαση παρατηρητη ειναι ανεξαρτητη της σχεδίασης του ελεγκτή δεν ειναι λαθος με βάση το σχημα 6.2 απο τις σημειωσεις του κεφ6?


Title: Re: [ΣΑΕ ΙΙ] Θέματα 2015
Post by: olgatsim on February 14, 2017, 17:12:35 pm
το (γ) που λεει στον ελεγχο ΓΧΑ συστηματων με αναδραση εξοδου και χρηση παρατηρητη, η σχεδίαση παρατηρητη ειναι ανεξαρτητη της σχεδίασης του ελεγκτή δεν ειναι λαθος με βάση το σχημα 6.2 απο τις σημειωσεις του κεφ6?

Αν διαβάσεις πάνω από το σχήμα λέει για την αρχή του διαχωρισμου. Λέει ότι τα Κ, L υπολογίζονται ανεξάρτητα το ένα από το άλλο. Ουσιαστικά το να σχεδιασουμε έναν ελεγκτή και έναν παρατηρητή σημαίνει να βρούμε αυτούς τους 2 πίνακες. Άρα σωστό το ερώτημα...


Title: Re: [ΣΑΕ ΙΙ] Θέματα 2015
Post by: DarkPassenger on February 14, 2017, 17:20:50 pm
Αν διαβάσεις πάνω από το σχήμα λέει για την αρχή του διαχωρισμου. Λέει ότι τα Κ, L υπολογίζονται ανεξάρτητα το ένα από το άλλο. Ουσιαστικά το να σχεδιασουμε έναν ελεγκτή και έναν παρατηρητή σημαίνει να βρούμε αυτούς τους 2 πίνακες. Άρα σωστό το ερώτημα...
ωραια ευχαριστώ!


Title: Re: [ΣΑΕ ΙΙ] Θέματα 2015
Post by: WatchDog on February 14, 2017, 17:26:24 pm
Πωωω σοβαρα?ο τύπος εβαλε σε Σ/Λ όλα σωστά??Πολύ hardcore η κατάσταση δηλαδή..


Title: Re: [ΣΑΕ ΙΙ] Θέματα 2015
Post by: DarkPassenger on February 14, 2017, 18:51:46 pm
Σεπτ 15 θέμα 3-β ποια ειναι μεθοδολογία/σκεπτική για την επίλυση του?


Title: Re: [ΣΑΕ ΙΙ] Θέματα 2015
Post by: iason1907 on February 14, 2017, 19:10:11 pm
Σεπτ 15 θέμα 3-β ποια ειναι μεθοδολογία/σκεπτική για την επίλυση του?
το βάζεις κοντά στο 0 (μηδέν δλδ) και βλέπεις τι γίνεται με τον έλεγχο. Επίσης από το προηγούμενο ερώτημα φαίνεται ότι αν πάρει τιμές κοντά στο μηδέν, από τις λύσεις που βγάζεις για τον ελεγκτή σου, δεν θα ήθελες το λ κοντά στο 0.

και τώρα που απάντησα για το γ, κρίμα να το σβήσω :p

Για το β, πάλι από τις λύσεις που έβγαλες φαίνεται ότι αρκεί να ελέγξεις την μία κατάσταση μόνο για να έχεις το επιθυμητό αποτέλεσμα. Το σύστημα παραμένει ελέγξιμο και ευσταθές. Εγώ έτσι το σκέφτηκα και φαίνεται να απλοποιείται ο ελεγκτής. Δεν παίρνω και όρκο  :D


Title: Re: [ΣΑΕ ΙΙ] Θέματα 2015
Post by: DarkPassenger on February 14, 2017, 19:18:07 pm
Για το β, πάλι από τις λύσεις που έβγαλες φαίνεται ότι αρκεί να ελέγξεις την μία κατάσταση μόνο για να έχεις το επιθυμητό αποτέλεσμα. Εγώ έτσι το σκέφτηκα και φαίνεται να απλοποιείται ο ελεγκτής. Δεν παίρνω και όρκο  :D

λογικη σκεψη μαλλον.. εχω βρει κ αυτή τη λύση.. πρεπει να ειναι απο παλιο ποστ οταν διαβαζα για σεπτέμβρη. εδω λεει βαζει γ, λ τετοια ωστε να μηδενίζει το Κ2


Title: Re: [ΣΑΕ ΙΙ] Θέματα 2015
Post by: heavy melon on February 14, 2017, 19:18:18 pm
Σεπτ. 2015, Θέμα 2ο, τελευταίο Σ-Λ
Στη σχεδίαση της ΓΑΚ η ελεγξιμότητα αποτελεί ικανή κ αναγκαία συνθήκη.
λάθος;
είναι ικανή αλλά όχι αναγκαία;;  :o


Title: Re: [ΣΑΕ ΙΙ] Θέματα 2015
Post by: iason1907 on February 14, 2017, 19:31:54 pm
λογικη σκεψη μαλλον.. εχω βρει κ αυτή τη λύση.. πρεπει να ειναι απο παλιο ποστ οταν διαβαζα για σεπτέμβρη. εδω λεει βαζει γ, λ τετοια ωστε να μηδενίζει το Κ2
το ίδιο έγραψα, στην λύση που ανέβασα άλλο έκανα :p


Title: Re: [ΣΑΕ ΙΙ] Θέματα 2015
Post by: iliasT on June 06, 2017, 17:39:54 pm
Σεπτέμβριος 2015 στο 2ο θέμα με τα Σ-Λ ξέρει κάποιος τι είναι το β; Και τα υπόλοιπα αν τα έχετε ευκολα δε θα με χαλούσαν :P



Title: Re: [ΣΑΕ ΙΙ] Θέματα 2015
Post by: DarkPassenger on June 07, 2017, 11:11:20 am
όταν ρωτάει αν το σύστημα είναι εύρωστο (π.χ. θέμα 1γ - Σεπτ 2015) τι ψάχνουμε να βρούμε? Αν είναι ασυμπτωτικά ευσταθές ή θέλει κάτι αλλο?


Title: Re: [ΣΑΕ ΙΙ] Θέματα 2015
Post by: princess_of_the_dawn on June 07, 2017, 11:39:05 am
όταν ρωτάει αν το σύστημα είναι εύρωστο (π.χ. θέμα 1γ - Σεπτ 2015) τι ψάχνουμε να βρούμε? Αν είναι ασυμπτωτικά ευσταθές ή θέλει κάτι αλλο?
++ γενικα η ευρωστία τί φα;


Title: Re: [ΣΑΕ ΙΙ] Θέματα 2015
Post by: Perasmus on June 07, 2017, 16:09:57 pm
όταν ρωτάει αν το σύστημα είναι εύρωστο (π.χ. θέμα 1γ - Σεπτ 2015) τι ψάχνουμε να βρούμε? Αν είναι ασυμπτωτικά ευσταθές ή θέλει κάτι αλλο?

+1


Title: Re: [ΣΑΕ ΙΙ] Θέματα 2015
Post by: Apostolof on June 07, 2017, 18:10:41 pm
όταν ρωτάει αν το σύστημα είναι εύρωστο (π.χ. θέμα 1γ - Σεπτ 2015) τι ψάχνουμε να βρούμε? Αν είναι ασυμπτωτικά ευσταθές ή θέλει κάτι αλλο?

Αν ξανακάνουμε τις πράξεις στη V' βγαίνει αν δε κάνω λάθος V'=-x1^2+x1v0 και επειδή v0>0 πρέπει x1<0. Δε φτάνει να πούμε ότι υπάρχει ευρωστία για x1<0;

Γενικά για την ευρωστία είχε πει ότι είναι το κατά πόσο εκτρέπομαι ή όχι από το προσδοκώμενο αποτέλεσμα. Σε παρόμοια παραδείγματα που έκανε έφτανε σε κάποιο περιορισμό όπως εδώ για τον οποίο δε χανόταν η ευστάθεια, σε αυτά τα παραδείγματα όμως είχαμε και ελεγκτή...


Title: Re: [ΣΑΕ ΙΙ] Θέματα 2015
Post by: DarkPassenger on June 07, 2017, 18:16:09 pm
Αν ξανακάνουμε τις πράξεις στη V' βγαίνει αν δε κάνω λάθος V'=-x1^2+x1v0 και επειδή v0>0 πρέπει x1<0. Δε φτάνει να πούμε ότι υπάρχει ευρωστία για x1<0;

Γενικά για την ευρωστία είχε πει ότι είναι το κατά πόσο εκτρέπομαι ή όχι από το προσδοκώμενο αποτέλεσμα. Σε παρόμοια παραδείγματα που έκανε έφτανε σε κάποιο περιορισμό όπως εδώ για τον οποίο δε χανόταν η ευστάθεια, σε αυτά τα παραδείγματα όμως είχαμε και ελεγκτή...
κάτι τέτοιο σκεφτόμουνα κ εγω..


Title: Re: [ΣΑΕ ΙΙ] Θέματα 2015
Post by: Apostolof on June 07, 2017, 18:35:13 pm
Σεπτέμριο '15 στο θέμα 3, το sqrt(2)/2 είναι το J;
Τα κέρδη βγαίνουν k1=4/λ & k2= 2sqrt(2)/λ - 4γ/λ;
Στο επόμενο ερώτημα επιλέγουμε γ=0 και λ=1;


Title: Re: [ΣΑΕ ΙΙ] Θέματα 2015
Post by: paul on June 07, 2017, 18:40:17 pm
όταν ρωτάει αν το σύστημα είναι εύρωστο (π.χ. θέμα 1γ - Σεπτ 2015) τι ψάχνουμε να βρούμε? Αν είναι ασυμπτωτικά ευσταθές ή θέλει κάτι αλλο?

Ειχε προταθει μια λυση αρκετες σελιδες πιο πισω...

Οπως εχεις το συστημα στο 1γ, ψαχνεις το σημειο ισορροπιας. βλεπεις οτι δεν ειναι το (0,0) και κανεις αλλαγη μεταβλητης.
Με την νεα μεταβλητη, οριζεις και μια Lyapunov, βρισκεις την παραγωγο της και με τα στοιχεια που σου δινει για το v0 την βγαζεις αρνητικα η ημιορισμενη. Κανεις Lassale, βλεπεις οτι το σημειο ισορροπιας ειναι ολικα ασυμπτωτικα ευσταθες.

Ετσι μαλλον καταληγεις στο οτι το συστημα ειναι ευρωστο.


Σεπτέμριο '15 στο θέμα 3, το sqrt(2)/2 είναι το J;
Τα κέρδη βγαίνουν k1=4/λ & k2= 2sqrt(2)/λ - 4γ/λ;
Στο επόμενο ερώτημα επιλέγουμε γ=0 και λ=1;

Κατι τετοια βρηκα κι εγω.
Πηρα ομως γ=λ=1. Μπορει να βγαινει και για γ=0. τσεκαρε το


Title: Re: [ΣΑΕ ΙΙ] Θέματα 2015
Post by: raptalex on June 07, 2017, 19:09:42 pm
Σεπτέμριο '15 στο θέμα 3, το sqrt(2)/2 είναι το J;
Τα κέρδη βγαίνουν k1=4/λ & k2= 2sqrt(2)/λ - 4γ/λ;
Στο επόμενο ερώτημα επιλέγουμε γ=0 και λ=1;

Ανεβάζεις λίγο μία φώτο με τις πράξεις σου;;


Title: Re: [ΣΑΕ ΙΙ] Θέματα 2015
Post by: iliasT on June 07, 2017, 19:15:29 pm
Σεπτέμριο '15 στο θέμα 3, το sqrt(2)/2 είναι το J;
Τα κέρδη βγαίνουν k1=4/λ & k2= 2sqrt(2)/λ - 4γ/λ;
Στο επόμενο ερώτημα επιλέγουμε γ=0 και λ=1;

Εγώ βρήκα k1 = 4/λ αλλά k2= (2*λ*sqrt(2) - 4γ)/λ^2
Μετά επιλέγω γ και λ έτσι ώστε k2 = 0 άρα βάζω γ = sqrt(2) = λ.

Edit: ανέβασα και μια φώτο με όλο το θέμα.


Title: Re: [ΣΑΕ ΙΙ] Θέματα 2015
Post by: DarkPassenger on June 07, 2017, 19:21:59 pm
Εγώ βρήκα k1 = 4/λ αλλά k2= (2*λ*sqrt(2) - 4γ)/λ^2
Μετά επιλέγω γ και λ έτσι ώστε k2 = 0 άρα βάζω γ = sqrt(2) = λ.

Edit: ανέβασα και μια φώτο με όλο το θέμα.

same here


Title: Re: [ΣΑΕ ΙΙ] Θέματα 2015
Post by: paul on June 07, 2017, 19:32:37 pm
Αλλαζω την απαντηση μου και συμφωνω με iliasT και DarkPassenger


Title: Re: [ΣΑΕ ΙΙ] Θέματα 2015
Post by: Luffy on June 07, 2017, 20:03:09 pm
Εγώ βρήκα k1 = 4/λ αλλά k2= (2*λ*sqrt(2) - 4γ)/λ^2
Μετά επιλέγω γ και λ έτσι ώστε k2 = 0 άρα βάζω γ = sqrt(2) = λ.

Edit: ανέβασα και μια φώτο με όλο το θέμα.


Συμφωνώ με τα αποτελέσματα σου αλλά έχω έναν ενδοιασμό για την επιλογή που έκανες για τους ενεργοποιητές. Στην εκφώνηση αναφέρει " η επιλογή των ενεργοποιητών να γίνει με σκοπό την απλούστευση του συστήματος".

Θεωρώ λοιπόν πως ένα σύστημα χωρίς καθόλου ενεργοποιητές (πιστεύω αυτό σημαίνει ότι είναι ίσοι με μηδέν) είναι πιο απλό. Κατά συνέπεια επιλέγω να μηδενίσω το γ και να επιλέξω μια τιμή για το λ που θα ανήκει στο R. Με αυτό τον τρόπο γλυτώνω την παρουσία ενός εκ των δύο ενεργοποιητών στο σύστημα και πετυχαίνω το ίδιο αποτέλεσμα.

Φιλικά πάντα.

ΥΓ: Όπως σχολίασα βέβαια δεν είμαι απολύτος σίγουρος πως η απουσία ενός ενεργοποιητή συνεπάγεται το μηδενισμό του.


Title: Re: [ΣΑΕ ΙΙ] Θέματα 2015
Post by: iliasT on June 07, 2017, 20:10:17 pm
Συμφωνώ με τα αποτελέσματα σου αλλά έχω έναν ενδοιασμό για την επιλογή που έκανες για τους ενεργοποιητές. Στην εκφώνηση αναφέρει " η επιλογή των ενεργοποιητών να γίνει με σκοπό την απλούστευση του συστήματος".

Θεωρώ λοιπόν πως ένα σύστημα χωρίς καθόλου ενεργοποιητές (πιστεύω αυτό σημαίνει ότι είναι ίσοι με μηδέν) είναι πιο απλό. Κατά συνέπεια επιλέγω να μηδενίσω το γ και να επιλέξω μια τιμή για το λ που θα ανήκει στο R. Με αυτό τον τρόπο γλυτώνω την παρουσία ενός εκ των δύο ενεργοποιητών στο σύστημα και πετυχαίνω το ίδιο αποτέλεσμα.

Φιλικά πάντα.

ΥΓ: Όπως σχολίασα βέβαια δεν είμαι απολύτος σίγουρος πως η απουσία ενός ενεργοποιητή συνεπάγεται το μηδενισμό του.

Mπορεί να είναι κι έτσι. Δεν είμαι σίγουρος τι ακριβώς σημαίνει η απλούστευση του συστήματος. Σίγουρα δε μπορείς να βάλεις το λ = 0 πάντως γιατί είναι στον παρονομαστή του k1. Δεν ξέρω αν το σύστημα είναι πιο απλό όταν λείπει το ένα k (k2) ή όταν λείπει ένας ενεργοποιητής (γ).


Title: Re: [ΣΑΕ ΙΙ] Θέματα 2015
Post by: Luffy on June 07, 2017, 20:24:37 pm
Mπορεί να είναι κι έτσι. Δεν είμαι σίγουρος τι ακριβώς σημαίνει η απλούστευση του συστήματος. Σίγουρα δε μπορείς να βάλεις το λ = 0 πάντως γιατί είναι στον παρονομαστή του k1. Δεν ξέρω αν το σύστημα είναι πιο απλό όταν λείπει το ένα k (k2) ή όταν λείπει ένας ενεργοποιητής (γ).

Ο ελεγκτής -σύστημα γενικότερα- θα ήταν πιο απλός αν παρέλειπες κάποιο από τα δύο κέρδη στη μελέτη σου εξολοκλήρου. Με το να μηδενίσεις κάποιο από τα κέρδη δεν το πετυχαίνεις.

Φιλικά

ΥΓ: Υπάρχει και αναφορά στις σημειώσεις του αποστόλοφ στην τελευταία παράδοση.
ΥΓ2: Έχεις δίκιο για το λ.


Title: Re: [ΣΑΕ ΙΙ] Θέματα 2015
Post by: s0r0n on June 07, 2017, 20:37:26 pm
Εχει κανεις τα Σ-Λ απο Σεπτ 15?


Title: Re: [ΣΑΕ ΙΙ] Θέματα 2015
Post by: iliasT on June 07, 2017, 20:54:12 pm
Εχει κανεις τα Σ-Λ απο Σεπτ 15?

Νομίζω όλα σωστά. Δεν είμαι σίγουρος για το β.


Title: Re: [ΣΑΕ ΙΙ] Θέματα 2015
Post by: tzitzikas1 on September 10, 2017, 18:52:44 pm
Σεπτέμβρης 2015 θεμα 3β

https://prnt.sc/gjg6bz

Ο πίνακας Β θεωρούμε ότι είναι γνωστός; Τα γ και λ δλδ είναι γνωστές σταθερές και πάμε κανονικά με γραμμική ανάδραση καταστάσεων;

Η είναι άγωνστα και παίρνουμε δυναμική;

edit: από ότι βλέπω στις προηγούμενες σελίδες θεωρούνται γνωστά και άρα πάω σε γραμμική .



Title: Re: [ΣΑΕ ΙΙ] Θέματα 2015
Post by: heavy melon on September 10, 2017, 19:56:40 pm
Ιουνης 2015, θεμα 4ο
από τα λυμένα
δε μου φαινεται πολυ λογικο αυτο με το (x1,x2,u) ως ΣΙ
εγω θα το εγραφα (x1*,x2*)=(sin^(-1)(cu*/a),0)
τι λέτε?

η λύση μου παρακάτω!


Title: Re: [ΣΑΕ ΙΙ] Θέματα 2015
Post by: heavy melon on September 14, 2017, 19:52:52 pm
Εγώ βρήκα k1 = 4/λ αλλά k2= (2*λ*sqrt(2) - 4γ)/λ^2
Μετά επιλέγω γ και λ έτσι ώστε k2 = 0 άρα βάζω γ = sqrt(2) = λ.

Edit: ανέβασα και μια φώτο με όλο το θέμα.


περί σεπτεμβρίου 2015, θέμα 3ο, ερώτημα β

πιο απλός γίνεται αν μηδενίσω το k2 όπως λέει  εδώ,
ή  αν δε βάλω και τις 2 καταστάσεις;
εγώ δηλ πίστευα ότι έπρεπε να μηδενίσω το γ
ώστε να έχω μία κατάσταση μόνο=πιο απλό ΣΚΒ
όχι?

exomag σώσε μας!


Title: Re: [ΣΑΕ ΙΙ] Θέματα 2015
Post by: heavy melon on September 14, 2017, 21:22:31 pm
περί σεπτεμβρίου 2015, θέμα 3ο, ερώτημα β

πιο απλός γίνεται αν μηδενίσω το k2 όπως λέει  εδώ,
ή  αν δε βάλω και τις 2 καταστάσεις;
εγώ δηλ πίστευα ότι έπρεπε να μηδενίσω το γ
ώστε να έχω μία κατάσταση μόνο=πιο απλό ΣΚΒ
όχι?

exomag σώσε μας!



Title: Re: [ΣΑΕ ΙΙ] Θέματα 2015
Post by: Alex_corfu on January 15, 2018, 21:13:13 pm
Έχει κανείς τη λύση για Φεβρουάριο 2015 θέμα 5 μήπως? Ευχαριστώ εκ των προτέρων!


Title: Re: [ΣΑΕ ΙΙ] Θέματα 2015
Post by: Endeavour X on January 20, 2018, 16:23:09 pm
Εεεε... σίγουρα; Εγώ το μόνο που βλέπω στην παρατήρηση εκείνη είναι ενα σχόλιο για την ελεγξιμότητα... Γενικά αν υπήρχε λυμένη αυτή η άσκηση απο ροβιθάκη, μάλλον θα ήταν σε άλλο κεφάλαιο, αυτό το θέμα δεν έχει σχέση με ελεγξιμότητα ( νομίζω δηλαδή  :P )

Edit: Τέλως πάντων, εγω θα ποσταρω αυτο που σκεφτηκα. Εμενα σωστο μου φαινεται, αλλα εγω ειμαι χαζος γενικα, οποτε πειτε και σεις...

Δινεται α>0 και β<0. Επισης ξερουμε οτι η συναρτηση lyapunov μπορει να ειναι οποιαδηποτε αρκει να ειναι θετικα ημιορισμενη σωστα; Επιλεγω την εξης:

V = 1/2α * χ12 - 1/2β * χ22 , η οποια ειναι θετικα ημιορισμενη διοτι β<0 και μαζι με το - που βαλαμε ειναι οκ...

Υπολογιζουμε την παραγωγο και μετα απο πραξεις εγω καταληγω σε αυτο:

 .
V = 1/αβ * ( βχ12 - αχ22 ) * ( 4 - χ12 - χ22 )

  • Το 1/αβ ειναι <0 παντα και παντου
  • Το βχ12 - αχ22 ειναι συνολικα <=0 ( =0 στο (0,0) ) γιατι αυτο ισχυει για καθε ορο, ειναι σαν αθροισμα τετραγωνων με ενα - απεξω
  • Ο τελευταιος ορος ειναι που αλλαζει προσημο αναλογα με το αν βρισκεται μεσα η εξω απο τον κυκλο

Οποτε ουσιαστικα εχουμε τις εξης περιπτωσεις

  • χ12 + χ22 < 4 . Τοτε η παραγωγος ειναι (-) * (-) * (+) = (+) οποτε οι καταστασεις θα κινουνται προς τον κυκλο μεχρι να τον φτασουν. Οταν φτασουν θα κινουνται πανω του.
  • χ12 + χ22 > 4 . Τοτε η παραγωγος ειναι (-) * (-) * (-) = (-) οποτε οι καταστασεις ξανα θα κινουνται προς τον κυκλο και οταν φτασουν θα κινουνται πανω του.

Με βαση αυτα καταληγω οτι ανεξαρτητα απο σημειο εκκινησης οι καταστασεις τελικα θα κινουνται πανω στον κυκλο με κεντρο (0,0) και ακτινα 2. Τι λετε;

Πως γινεται η παραγωγος της V να ειναι θετικη και οι λυσεις να συγκλινουν πανω σε μια τροχια?


Title: Re: [ΣΑΕ ΙΙ] Θέματα 2015
Post by: princess_of_the_dawn on January 21, 2018, 01:06:32 am
Πως γινεται η παραγωγος της V να ειναι θετικη και οι λυσεις να συγκλινουν πανω σε μια τροχια?

Η παράγωγος μας δείχνει τη μεταβολή της συνάρτησης, συγκεκριμένα η χρονική παράγωγος τί συμβαίνει στη συνάρτηση με το πέρασμα του χρόνου. Θετική παράγωγος σημαίνει ότι με το χρόνο η συνάρτηση θα αυξάνει. Παράγωγος μηδεν σημαίνει καμία μεταβολή. Δηλαδή όταν η παράγωγος μηδενιστεί η συνάρτηση δε θα μεταβάλλεται άλλο. Η παράγωγος μηδενίζεται για x12+x22=4 άρα ξεκινώντας εσωτερικά του κύκλου η συνάρτηση είναι θετική άρα αυξάνει μέχρι να φτάσει στον εν λόγω κύκλο, όποτε και θα μείνει εκεί για πάντα πάντα πάντα, αφού είναι το μεγαλύτερο αμετάβλητο σύνολο που μηδενίζει την παράγωγο(LaSalle).


Title: Re: [ΣΑΕ ΙΙ] Θέματα 2015
Post by: Maylo on June 05, 2018, 13:16:11 pm
Καλησπέρα , για το θέμα 4 ερώτημα (γ) Ιούνιος 2015  έχουμε κάποια ιδέα για το αν παίρνουμε δυναμική ή γραμμική ανάδραση; Το 'χετε λύσει; :o :o


Title: Re: [ΣΑΕ ΙΙ] Θέματα 2015
Post by: kanou_tom on June 05, 2018, 13:27:58 pm
Καλησπέρα , για το θέμα 4 ερώτημα (γ) Ιούνιος 2015  έχουμε κάποια ιδέα για το αν παίρνουμε δυναμική ή γραμμική ανάδραση; Το 'χετε λύσει; :o :o

Εγώ προσωπικά το έλυσα με δυναμική ανάδραση.. Μου προέκυψε ένα ΣΚΒ με πίνακα Α 3Χ3, βρήκα το χαρακτηριστικό πολυώνυμο και μετά χρησιμοποίησα κριτήριο Routh-Hurwitz για να βγάλω τα κέρδη έτσι ώστε το σύστημα να είναι ευσταθές


Title: Re: [ΣΑΕ ΙΙ] Θέματα 2015
Post by: Maylo on June 05, 2018, 13:34:04 pm
Εγώ προσωπικά το έλυσα με δυναμική ανάδραση.. Μου προέκυψε ένα ΣΚΒ με πίνακα Α 3Χ3, βρήκα το χαρακτηριστικό πολυώνυμο και μετά χρησιμοποίησα κριτήριο Routh-Hurwitz για να βγάλω τα κέρδη έτσι ώστε το σύστημα να είναι ευσταθές

Χαρακτηριστικό πολυώνυμο = s^3 + s^2 (b+c*k2) + s (acosx1* + c*k1 + c*ki)   ;;;;


Αυτό βρήκα εγώ , και για Pd(s) = ....  βρίσκω κάποιες συνθήκες για k2=(a1-b)/c  και  (ki+k1)= ...  , συν ο περιορισμός για θετικό συντελεστη από Routh-HUrwitz . Μετά πώς το δουλεύουμε ;


Title: Re: [ΣΑΕ ΙΙ] Θέματα 2015
Post by: kanou_tom on June 05, 2018, 13:54:23 pm
Χαρακτηριστικό πολυώνυμο = s^3 + s^2 (b+c*k2) + s (acosx1* + c*k1 + c*ki)   ;;;;


Αυτό βρήκα εγώ , και για Pd(s) = ....  βρίσκω κάποιες συνθήκες για k2=(a1-b)/c  και  (ki+k1)= ...  , συν ο περιορισμός για θετικό συντελεστη από Routh-HUrwitz . Μετά πώς το δουλεύουμε ;

Από το κριτήριο Routh-Hurwitz βγάζεις κάποιους περιορισμούς για τα κέρδη του ελεγκτή σου. Ουσιαστικά, έτσι επιτυγχάνεις ότι οι ιδιοτιμε΄ς του συστήματος θα έχουν όλες αυστηρά αρνητικό πραγματικό μέρος. Πετυχαίνοντας αυτό, το σύστημα οδηγείτε στην ασυμπτωτική ευστάθεια και συνεπώς w_dot = y - δ = 0 άρα y = δ , που είναι και αυτό που θες.

Επειδή δεν ξέρω αν γίνομαι κατανοητός, μπορείς να δεις τις διαφάνειες του Ροβι για την δυναμική ανάδραση


Title: Re: [ΣΑΕ ΙΙ] Θέματα 2015
Post by: AB_Kingbird on June 07, 2018, 18:46:23 pm
Ορίστε και η προσέγγιση μου στο Θέμα 3 του Σεπτέμβρη 2015.



Title: Re: [ΣΑΕ ΙΙ] Θέματα 2015
Post by: κύριος Φασόλης on June 07, 2018, 20:15:36 pm
Στο τελευταιο θεμα του Φεβ15 θετω απλα καταλληλο u ετσι ωστε το συστημα που μου δινει να ερθει στη κλασικη μορφη του ελατηριου αποσβεστηρα;


Title: Re: [ΣΑΕ ΙΙ] Θέματα 2015
Post by: princess_of_the_dawn on June 07, 2018, 20:25:47 pm
Στο τελευταιο θεμα του Φεβ15 θετω απλα καταλληλο u ετσι ωστε το συστημα που μου δινει να ερθει στη κλασικη μορφη του ελατηριου αποσβεστηρα;
ακριβώς βλέπεις πώς το θες και κόβεις/ράβεις


Title: Re: [ΣΑΕ ΙΙ] Θέματα 2015
Post by: pesto80 on August 03, 2018, 15:06:42 pm
Ιουνιος 15
θεμα 1ο
Ε. σωστο δεν ειναι?
επειδη μια κατασταση ειναι  ενα σημειο (τιμη ) του χωρου καταστασεων?
ενω η ταξη ειναι το πληθος τον εξισωσεων καταστασης?
λαθος ειναι οσες εξισωσεις καταστασης εχεις τοσες μεταβλητες καταστασης εχει. Επισης, οχι οτι αποδεικνυει κατι για το συγκεκριμενο ερωτημα, απλα επειδη αναφερθηκαν οι εξισωσεις καταστασης για να ειναι ελαχιστης υλοποιησης δηλαδη να εκφραζεται το συστημα με τις λιγοτερες δυνατες μεταβλητες πρεπει να ειναι ελεγξιμο και παρατηρησιμο

υπαρχει καποιος με πιο καίρια αποψη; νομιζω οτι ο φιλος ρφεντ ειχε δικαιο στην διαπιστωση του. το ρωταω ωστοσο γιατι και στα λυμενα που βρηκα το ειχε δυστυχως  ως "λαθος". Το επιχειρημα μου ειναι οτι ο ροβι απαιτει να διαλεγουμε οσες μεταβλ οσο ειναι η ταξη, ωστοσο τις μεταβλητες τις ΕΠΙΛΕΓΟΥΜΕ. Συνεπως μπορει υπαρχουν περισσοτερες διαθεσιμες μεταβλητες απο όσο ειναι η ταξη. Και το επιχειρημα μου ειναι εγκυρο μονο στην περιπτωση που ο ορος "κατασταση" ταυτιζεται με τον ορο "μεταβλητη καταστασης" που νομιζω οτι ειναι λαθος. Οποτε σε εκεινη την περιπτωση ο ρφεντ το εθεσε καλυτερα.


Title: Re: [ΣΑΕ ΙΙ] Θέματα 2015
Post by: mprizou on August 03, 2018, 15:31:12 pm
ειμαι αρκετα σιγουρη πως ο ορος κατασταση αναφερεται σε μια τιμη (νουμερο), επομενως οντως υπαρχουν απειρες καταστασεις. σε αντιστοιχη ερωτηση για τις μεταβλητες καταστασης θα ηταν οντως "λαθος" η προταση


Title: Re: [ΣΑΕ ΙΙ] Θέματα 2015
Post by: pesto80 on September 07, 2018, 21:29:28 pm
συγγνωμη φεβρουαριος 2015 θεμα 5 ( http://prntscr.com/krubj5 )ειδα να προτεινεται αυτη η λυση: http://prntscr.com/kruboe

ειμαστε σιγουροι οτι ειναι αυτη η σωστη απαντηση? δηλαδη απλα αναιρεσε ολους τους ορους με αντιστροφες πραξεις (πρσθεσεις-αφαιρεσεις, πολλαπλασιασμους-διαιρεσεις) και εβαλε αυτα που ηθελε στην εισοδο του ωστε με καταλληλες πραξεις να καταληγει το επιθυμητο συστημα. ειναι δυνατον να ζητησε κατι τοσο απλο ο ροβι?


Title: Re: [ΣΑΕ ΙΙ] Θέματα 2015
Post by: MrRobot on September 08, 2018, 02:27:44 am
Σωστή είναι η λύση. Το έκανε και στο μάθημα φέτος, το ίδιο παράδειγμα. Επίσης το πρώτο θέμα του Ιουνίου του 2018 ήταν στην ίδια λογική.

Γενικά ζητάει διάφορα πράγματα αλλά μη πηγαίνεις έχοντας στο μυαλό σου πάντα το πιο δύσκολο, κάποια θέματα είναι απλά.


Title: Re: [ΣΑΕ ΙΙ] Θέματα 2015
Post by: pesto80 on September 08, 2018, 13:54:09 pm
ευχαριστω πολυ ρε.

μια αλλη ερωτηση που ηθελα να κανω ειναι κατα ποσο ειναι σωστο ή θεμιτο σε αυτην την ασκηση ( http://prntscr.com/ks0yo0 ) να παρουμε ως εξισωσεις καταστασεις x1=x x2=x' και x3= x'' με το σκεπτικο οτι εχουμε να κανουμε με συστημα 3ης ταξης, αντι για x3= i. και επειτα με σωστη απαλοιφη του i να σχηματισω τις εξισωσεις καταστασεις


Title: Re: [ΣΑΕ ΙΙ] Θέματα 2015
Post by: Exomag on September 08, 2018, 14:32:51 pm
ευχαριστω πολυ ρε.

μια αλλη ερωτηση που ηθελα να κανω ειναι κατα ποσο ειναι σωστο ή θεμιτο σε αυτην την ασκηση ( http://prntscr.com/ks0yo0 ) να παρουμε ως εξισωσεις καταστασεις x1=x x2=x' και x3= x'' με το σκεπτικο οτι εχουμε να κανουμε με συστημα 3ης ταξης, αντι για x3= i. και επειτα με σωστη απαλοιφη του i να σχηματισω τις εξισωσεις καταστασεις

Είναι valid λύση επίσης, εφόσον έβγαζες σωστά τις εξισώσεις κατάστασεις δε θα είχες θέμα.


Title: Re: [ΣΑΕ ΙΙ] Θέματα 2015
Post by: pesto80 on September 12, 2018, 19:45:29 pm
Συμφωνώ με τα αποτελέσματα σου αλλά έχω έναν ενδοιασμό για την επιλογή που έκανες για τους ενεργοποιητές. Στην εκφώνηση αναφέρει " η επιλογή των ενεργοποιητών να γίνει με σκοπό την απλούστευση του συστήματος".

Θεωρώ λοιπόν πως ένα σύστημα χωρίς καθόλου ενεργοποιητές (πιστεύω αυτό σημαίνει ότι είναι ίσοι με μηδέν) είναι πιο απλό. Κατά συνέπεια επιλέγω να μηδενίσω το γ και να επιλέξω μια τιμή για το λ που θα ανήκει στο R. Με αυτό τον τρόπο γλυτώνω την παρουσία ενός εκ των δύο ενεργοποιητών στο σύστημα και πετυχαίνω το ίδιο αποτέλεσμα.

Φιλικά πάντα.

ΥΓ: Όπως σχολίασα βέβαια δεν είμαι απολύτος σίγουρος πως η απουσία ενός ενεργοποιητή συνεπάγεται το μηδενισμό του.

  Υπαρχει τελεσιδικη απαντηση εκ μερους του τημμυ εν τελει? στις λυμενες απλοποιηση θεωρει τους οσο το δυνατον λιγοτερους συντελεστες κερδους (κοινως τα k). πραγμα το οποιο φαινεται και σε μενα λογικο καθως θα αναγκαστουμε να προσθεσουμε λιγοτερη "επιδραση" λιγοτερο "ζορι" στο συστημα προκειμενου να κανει αυτο που θελουμε. Εν τελει ο φιλος λαφυ λεει οτι απλοποιηση εστι μηδενισμος ενεργοποιητων. Τελικα τι;

  ΥΓ: Επισης πολυ σημαντικο. Στην περιπτωση που βρισκουμε ενα σημειο ισορροπιας στη μορφη (χ1*,χ2*,u*)=(0,x2*, a-sinx2*/c) ξερω γω. Ποσο σωστο ειναι αυτο? (βλεπε θεμα 4 ιουνου 15). Ποσο επιτρεπτο ειναι να θεσω οτι το χ2* που δεν εχω καμια προσθετη εξισωση για να το προσδιορισω, θα ειναι μια "ταδε" τιμη?


Title: Re: [ΣΑΕ ΙΙ] Θέματα 2015
Post by: unfo on September 13, 2018, 09:44:16 am
Σεπτέμβρης15-θεμα1 στις λύσεις στο β ερώτημα λέει οτι είναι ΟΛΙΚΑ ασυμπωτικά ευσταθές, λόγω του περιορισμού στο γ (γ>0) δεν θα έπρεπε να είναι ΤΟΠΙΚΆ  ασυμπτωτικα ευσταθές από LaSalle ?


Title: Re: [ΣΑΕ ΙΙ] Θέματα 2015
Post by: leukosaraphs! on September 13, 2018, 09:50:35 am
Σεπτέμβρης15-θεμα1 στις λύσεις στο β ερώτημα λέει οτι είναι ΟΛΙΚΑ ασυμπωτικά ευσταθές, λ΄΄ογω του περιορισμού στο γ (γ>0) δεν θα έπρεπε να είναι ΤΟΠΙΚΆ  ασυμπτωτικα ευσταθές από LaSalle ?

στον "τυπο" της ευσταθειας, παιζουν ρολο οι περιορισμοι που εσυ εισαγεις στις μεταβλητες καταστασης!


Title: Re: [ΣΑΕ ΙΙ] Θέματα 2015
Post by: unfo on September 13, 2018, 10:00:08 am
στον "τυπο" της ευσταθειας, παιζουν ρολο οι περιορισμοι που εσυ εισαγεις στις μεταβλητες καταστασης!

 Ααααα δηλαδή μόνο τους δικούς μας περιορισμούς και όχι αυτούς που δίνει η άσκηση σε κάποιες μεταβλητές της, μάλιστα, είσαι ωραίος !!!


Title: Re: [ΣΑΕ ΙΙ] Θέματα 2015
Post by: leukosaraphs! on September 13, 2018, 10:05:47 am
Ααααα δηλαδή μόνο τους δικούς μας περιορισμούς και όχι αυτούς που δίνει η άσκηση σε κάποιες μεταβλητές της, μάλιστα, είσαι ωραίος !!!

ο τοπικος χαρακτηρας βγαινει απο το γεγονος οτι δεν σου "βγαινουν" τα μαθηματικα και κανεις (συνηθως) Taylor για να δειξεις τα πραγματα που θες (πχ. sin(x) = x, για μικρα x) . Οταν η ασκηση σου λεει οτι οι μεταβλητες καταστασης σου ανηκουν στο X υποσυνολο του Rn ... τοτε η ολικη σου ευσταθεια αναφερεται σε αυτο το συνολο X, δεν ορισες εσυ μια γειτονια κλπ.


Title: Re: [ΣΑΕ ΙΙ] Θέματα 2015
Post by: unfo on September 13, 2018, 10:40:17 am
Μου τις έλυσες όλες τις απορίες, ευχαριστώ πολύ ! :)


Title: Re: [ΣΑΕ ΙΙ] Θέματα 2015
Post by: Florence on September 13, 2018, 19:48:01 pm
Τελικα οταν ζηταει απλοποιηση συστηματος κλειστου βρογχου προσπαθουμε να ελαχιστοποιησουμε τα κερδη(k1,k2) ή τους ενεργοποιητές (στοιχεια του B)???????


Title: Re: [ΣΑΕ ΙΙ] Θέματα 2015
Post by: pesto80 on June 08, 2019, 13:43:53 pm
Ιουν 15, Θεμα 1.γ). Είναι λάθος επειδή λέει κέρδη ανάδρασης; Δηλαδη k1, k2 κτλ χωρίς να συμπερλιμβάνεται σε αυτά (τα κέρδη ανάδρασης) και το kr δηλαδη (το οποίο οδηγεί την τιμή της εξόδου);


Title: Re: [ΣΑΕ ΙΙ] Θέματα 2015
Post by: Spiro on June 08, 2019, 14:20:03 pm
Είναι λάθος γιατί δεν ξέρεις αν είναι ελέγξιμο. Ελεγκτή πάντα μπορείς να φτιάξεις (δεδομένου ότι οι ΜΚ σου είναι μετρήσιμες). Αν όμως το σύστημα σου δεν είναι ελέγξιμο, τότε ξέρεις πως όποιον ελεγκτή και να χρησιμοποιήσεις η έξοδος δεν μπορεί να πάει σε επιθυμητή σταθερή τιμή.


Title: Re: [ΣΑΕ ΙΙ] Θέματα 2015
Post by: pesto80 on June 08, 2019, 14:21:11 pm
Το kr ειναι κερδος αναδρασης η οχι;


Title: Re: [ΣΑΕ ΙΙ] Θέματα 2015
Post by: Spiro on June 08, 2019, 14:30:52 pm
Μην κολλάτε στις λέξεις ρε σεις.. Αλλάζει κάτι αν θα το λέω κέρδος ανάδρασης ή απλό κέρδος; Δεν είναι εκεί το λάθος της πρότασης.


Title: Re: [ΣΑΕ ΙΙ] Θέματα 2015
Post by: ζεν on July 02, 2020, 21:03:45 pm
Ιουνιος 2015 στα σλ το τελευταιο ειναι σωστο? διοτι οταν λεει καταστασεις εννοει τις τιμες και οχι τις μεταβλητες καταστασεις? ενω αν ελεεγε μεταβλητες τοτε θα ήταν λάθος;


Title: Re: [ΣΑΕ ΙΙ] Θέματα 2015
Post by: Thunderlord on July 02, 2020, 21:06:06 pm
Ιουνιος 2015 στα σλ το τελευταιο ειναι σωστο? διοτι οταν λεει καταστασεις εννοει τις τιμες και οχι τις μεταβλητες καταστασεις? ενω αν ελεεγε μεταβλητες τοτε θα ήταν λάθος;

Δες εδώ (https://www.thmmy.gr/smf/index.php?topic=72032.0)

Και γενικά για τα θέματα των ΣΑΕ 2 δείτε τα τοπικς της κάθε χρονιάς, είναι πολύ καλά οργανωμένα


Title: Re: [ΣΑΕ ΙΙ] Θέματα 2015
Post by: ζεν on July 02, 2020, 21:07:21 pm
το εχω δει ολο το topic και διαφωνουν καποιο λενε σωστο καποιοι λενε λαθος γι αυτο ρωτησα


Title: Re: [ΣΑΕ ΙΙ] Θέματα 2015
Post by: Thunderlord on July 02, 2020, 21:30:57 pm
το εχω δει ολο το topic και διαφωνουν καποιο λενε σωστο καποιοι λενε λαθος γι αυτο ρωτησα

Ναι το ξέρω, απλά ρώτα εκεί για να μείνει στην κατάλληλη χρονιά  ;)

Εγώ προσωπικά θεωρώ ότι είναι λάθος, αν και tricky


Title: Re: [ΣΑΕ ΙΙ] Θέματα 2015
Post by: ζεν on July 02, 2020, 21:51:22 pm
Ναι το ξέρω, απλά ρώτα εκεί για να μείνει στην κατάλληλη χρονιά  ;)

Εγώ προσωπικά θεωρώ ότι είναι λάθος, αν και tricky

μα καταστασεις ειναι οι τιμες οχι οι μεταβλητες καταστασης ..


Title: Re: [ΣΑΕ ΙΙ] Θέματα 2015
Post by: Arcade on July 08, 2020, 06:54:02 am
Ιουνιος 2015 στα σλ το τελευταιο ειναι σωστο? διοτι οταν λεει καταστασεις εννοει τις τιμες και οχι τις μεταβλητες καταστασεις? ενω αν ελεεγε μεταβλητες τοτε θα ήταν λάθος;

Γνώμη μου: είναι σωστό γιατί μπορώ να διαλέξω εγώ τι μεταβλητές κατάστασης(=καταστάσεις) θα πάρω και ανάλογα με τους συνδυασμούς που θα κάνω τα χ' θα εκφράζονται με διαφορετικούς συνδυασμούς (πάντα οι συνδυασμοί θα έχουν μέγεθος ν, αλλά οι συνδυασμοί μπορεί να είναι περισσότεροι από ν)


Title: Re: [ΣΑΕ ΙΙ] Θέματα 2015
Post by: Valaam on September 08, 2020, 16:43:35 pm
Στο pdf με τις λύσεις Ιουνιου 15 θεμα 4γ:
γιατι εχει δυναμικο ελεγκτη ενω ζηταει γραμμικο;


Title: Re: [ΣΑΕ ΙΙ] Θέματα 2015
Post by: pesto80 on September 08, 2020, 17:22:47 pm
Στο pdf με τις λύσεις Ιουνιου 15 θεμα 4γ:
γιατι εχει δυναμικο ελεγκτη ενω ζηταει γραμμικο;

με τον κλασικο "γραμμικο"  ελεγκτη v= -k1*z1 - k2*z2 μπορείς να ελέγξεις τον ρυθμό με τον οποίο θα φτάσουμε στην επιθυμητή έξοδο (μέσω των k's), όχι όμως την επιθυμητή έξοδο αυτή καθ'αυτή.

hint: τσέκαρε στα ίδια θέματα το 1.γ. από τα Σ/Λ ;)

Ωστόσο με λίγο ψάξιμο στη θεωρία μπορούμε να βρούμε ότι με τον πλήρη γραμμικό ελεγκτή u = -k*x - k_r*r μπορούμε όντως να ελέγξουμε την έξοδο, μέσω του kr. H λυση του φίλου είναι η αλήθεια χρησιμοποιεί ολοκληρωτή ο οποίος χρησιμοποιείται όταν:

1. θέλουμε να εξαλείψουμε διαταραχές
2. το σύστημα έχει ασάφειες
3. υπάρχει περιορισμός στα κέρδη (k)

... οπότε όντως και εγώ δεν συμφωνώ με την επιλογή του συγκεκριμένου ελεγκτή.

ps: δεν έχω κάτσει να δω αν λύνεται με k_r ωστόσο


Title: Re: [ΣΑΕ ΙΙ] Θέματα 2015
Post by: Valaam on September 08, 2020, 18:24:15 pm
Quote
με τον κλασικο "γραμμικο"  ελεγκτη v= -k1*z1 - k2*z2 μπορείς να ελέγξεις τον ρυθμό με τον οποίο θα φτάσουμε στην επιθυμητή έξοδο (μέσω των k's), όχι όμως την επιθυμητή έξοδο αυτή καθ'αυτή.

hint: τσέκαρε στα ίδια θέματα το 1.γ. από τα Σ/Λ ;)
γιατι να μην παω ομως με τον -kx-kr*r?


Title: Re: [ΣΑΕ ΙΙ] Θέματα 2015
Post by: pesto80 on September 08, 2020, 18:29:27 pm
γιατι να μην παω ομως με τον -kx-kr*r?

... με πρόλαβες. Συμφωνώ εν τέλει.


Title: Re: [ΣΑΕ ΙΙ] Θέματα 2015
Post by: Soulaki on September 10, 2020, 00:54:51 am
... με πρόλαβες. Συμφωνώ εν τέλει.

Έτσι όμως βγαίνει kr=-c το οποίο είναι άγνωστο άρα δεν μπορούμε να σχεδιάσουμε τον ελεγκτή
sent from mTHMMY (https://play.google.com/store/apps/details?id=gr.thmmy.mthmmy) 


Title: Re: [ΣΑΕ ΙΙ] Θέματα 2015
Post by: Soulaki on September 10, 2020, 01:12:25 am
Στο pdf με τις λύσεις Ιουνιου 15 θεμα 4γ:
γιατι εχει δυναμικο ελεγκτη ενω ζηταει γραμμικο;

Νομίζω οτι εφόσον θέλει σταθεροποίηση στην έξοδο που είναι το θ=x1 του αρχικού συστήματος επιλέγουμε χ1*=δ ως το σ.ι άρα έτσι στο γραμμικοποιημενο θα ζητάμε z1->0 το οποίο πετυχαινουμε με απλό ελεγκτή -kz. Για τα k βγαίνουν περιορισμοί βάσει των ρ1,ρ2 που θεωρούνται γνωστά.
sent from mTHMMY (https://play.google.com/store/apps/details?id=gr.thmmy.mthmmy)  


Title: Re: [ΣΑΕ ΙΙ] Θέματα 2015
Post by: Nickgian on April 08, 2021, 17:50:51 pm
Υπαρχουν πουθενα η λυση για το θεμα 5 φεβ. 2015


Title: Re: [ΣΑΕ ΙΙ] Θέματα 2015
Post by: vterz on April 08, 2021, 21:16:15 pm
Υπαρχουν πουθενα η λυση για το θεμα 5 φεβ. 2015

Είναι η άσκηση που παρουσιάζεται στις σελίδες 52-53 στις σημείωσεις του Καναβούρα αν δε κάνω λάθος
Γενικά την κάνει και στο μάθημα και μάλλον θα είναι στα βιντεάκια που θα ανεβάσει στην πορεία του μαθήματος